Вы находитесь на странице: 1из 966
Complete Solutions Guide for CALCULUS EIGHTH EDITION Larson / Hostetler / Edwards Volume I Chapters P-6 Bruce H. Edwards University of Florida Houghton Mifflin Company Boston New York Copyright © 2006 by Houghton Mifflin Company. All rights reserved. Houghton Mifflin Company hereby grants you permission to reproduce the Houghton Mifflin material contained in this work in classroom quantities, solely for use with the ‘accompanying Houghton Mifflin textbook. All reproductions must include the Houghton Miffin copyright notice, and no fee may be collected except to cover the cost of duplication, If you wish to make any other use of this material, including reproducing or transmitting ‘the material or portions thereof in any form or by any electronic or mechanical means, including any information storage or retrieval system, you must obtain prior written permission from Houghton Mifflin Company unless such use is expressly permitted by federal copyright law. If you wish to reproduce material acknowledging a rights holder ‘ther than Houghton Mifflin Company, you must obtain permission from the rights holder, ‘Address inquiries to: College Permissions, Houghton Mifflin Company, 222 Berkeley Street, Boston, MA 02116-3764, Printed in the USS.A. ISBN: 0-618-52793-1 123456789- EB -09 08 07 06 0S CONTENTS Chapter P Preparation for Calculus. ............00005 Chapter 1 Limits and Their Properties ..............5+ Chapter 2 Differentiation Chapter 3. Applications of Differentiation.............. Chapter 4 Integration .. Chapter 5 Logarithmic, Exponential, and Other Transcendental Functions . . . . 544 Chapter 6 Differential Equations . .. 440 CHAPTER P Preparation for Calculus Section P:l Section P.2 Section P3 Section P4 Problem Solving . Graphs and Models. . . « Linear Models and Rates of Change... « Functions and Their Graphs... .. Fitting Models to Data... 2.0 ose CHAPTER P Preparation for Calculus Section P.1 Graphs and Models . dx +2 Rye Ve intercept: (4,0) intercepts: (—3,0), (3, 0) intercept: (0,2) yrinteroept: (0,3) Matches graph b). ‘Matches graph (2). Bynanat Ayeor intercepts: (2,0), (~2,0) aeinterepts: (0,0), (=1, 0), (1,0) srintrcept: (0, 4) ‘intercept: (0,0) Matches graph Matches graph (9) Byeketl 6 y= 6-2 x] -4]-2Jo[2 [4 x] [-Jeo[+[2[3 y[-s[2h; [4 [7 y[o[s [els[2fo Section PJ Graphs and Models 2/3 1[2 4 7’ 1]2]3 2)3]5 ADE EE 16, 4 when x = 0. Ximin = 30 Xmax = 30 Xsel=5 Ymin =-10 Ymnax = 40 Yscl=5 Note that y = 10 when x = Oorx = 10. 3 4 Chapter P__ Preparation for Calculus: nye SF yew se @ @»)=@173) (y= V5=3= V3= 1.73) (@) (-05,y) = (05, 2.47) © &3I=(-43) B= VI=T4) 1 ya tx-2 srimercept: y= 0? 40-2 y= =2;0,-2) intercepts: 0 = 8 +x ~ O= (+ 2)e-1) 2,1; (-2,0), (1,0) 2 y= 8 VBoF ye OVBTE y= 04(0,0) intercepts: 0 = 2 JB F 0= 2E= ETH = 0,45; 0,0) (45,0) intercept: pny -82=vi intercept None. xeannot equal 0 intercept: 28, vty + Ay imterepe: ©) — 8 + ay ¥=0;(0,0) seinercept: 200) — 8 + 40) = 0 x= 0; (0,0) ©) (3, —4) = (1.65, ~4) and (x, —4) = (1, -4) 20, y= 39 — 4x spintercept: y# = 0° 4(0) ¥=0;0,0) svlntercepts: 0 = 2? ~ 4x = xe - De +2) 0,42; (0,0), (42,0) 5 (0, -1) VET 3 (1,0) 243 wy GE _ & +30) IESE Y = BO) + IF y= 0;(0,0) on Bt Sime: OG rE e+) OG iE £=0,-3:(0,0), (~3,0) 26. y= 2x = JETT ysintercept: y = 2(0) - FFT y=-b@-1) intercept 0 = 2x- VFFT dee PFT ateett ate 1 “3 3 eat -4(4 we Note: x = ~ J/3/3 is an extraneous solution, 2 yaat—x [No symmetry with respect to either axis or the origin. 30, Symmetric with respect tothe origin since (9) = P+) nya nea yexte ‘32. Symmettie with respect tothe x-axis since 10. x(-yP =? = ‘34, Symmetric with respect tothe origin since (-2)(-») - J4= CaP = 0 xy- VIB = 0. ji leet wit pect ys 2 sincey = “ped eT 38, |y| ~ x = 3 is symmetric with respect tothe x-axis since |[-y| = x= 3 bl -x=3. Section P.l__Graphs and Models 27, Symmetric with respect to the yaxis since na =D t= 2 29, Symmetric with respect tothe a-axis since (oP =p a = de ‘31, Symmetric with respect to the origin since CaCy) aay = 4. By 4— VETS [No symmetry with respect to either axis or the origin. 38. Symmetric with respect tothe origin since 37. y = [2° + 2] is symmetric with respect tothe yraxis (—aP + (=2)| = [-@? + 2)] = be + at since y = 30, y= ne +2 Intercepts: G.0), 0,2) ‘Symmetry: none 5 6 Chapter P__ Preparation for Caleulus a2 40, ay aayadeet Intercepts: Intercepts: Intercepts: (4,0),(0,2) 6.0, 0,4) (.0,(-40) Symmetry: none Symmetry: none Seeaeiecee By Heyes 45. y= (e+ 3P Intercepts: Intercept: 0,3) Intercepts: (1,0), (-1,0), 0,1) ‘Symmeny: y-axis (-3,0),(0,9) ‘Symmetry: axis Symmetry: none wae, 46, y= 2s? + x= xe + 1) a yaP +d 4 yaar Intercepts: Intercepts: Intercepts: (0.0), (4,0) (- %.0),00.2) (0.0), @,0),(-2,0) ‘Symmetry: none ‘Symmetry: none ‘Symmetry: origin 9. y= OE ; Intercepts: Intercepts : 0,0), (-2,0) (3,0),6,0), (0,3) a9 Symmetry: none Domain: x 2 -2 Symmetry: y-axis Domain: [3,3] __10 FT shy Intercept: (0, 10) Symmetry: y-axis eho 51. yo x vax yove td Intercepts: (0,3), ©, ~3), (~9, 0) Symmetry: axis 60, 3x ~ 4y? = 8 4y? = 3 yeave-2 Intereept: (§,0) Symmetry: xaxis Section P.1__ Graphs and Models Saxe yn 4 Intercepts: Intereepts: none (0.2, ,-24(-40) ees Symmetry: ais ; 58. y= 6— bl 56. y=|6~ a] Intercepts Intercepts: (0.8,(-6.0),6,0) (0.9.6.0) Symmetry: yas Symmetry: none 58244 = dye Intercepts: (2,0), 2,0), (0,-1),(0,1) ro Symmetry: origin and both axes Intercepts: Domain: [-2,2] 6,0), (0, v2),(0, Symmetry: xaxis [eA | eo}, eA) "The coresponding y-vale is y = 1. Point of intersection: (1, 1) 2 8 Chapter P__ Preparation for Calculus @. x= Baye BSH = Se Set3yeloy= 2e=13_ 1-5 3 2-3 Ik 1-5 “ xe2 ‘The corresponding y-valc is y Point of intersection: (2, ~3) 6 x=3-yay yernl 3-x=@-1F Boxes -20t1 Om t= x-2= (e+ Dhe-2) ‘The corresponding y-values are y = —2 and y Points of intersection: (—1, ~2),(2,1) 66, 2 += Baya Is—F 2x ty=1W=>y = 10-2 25 ~ x2 = (10 — 23)? 25 ~ x2 = 100 - 40x + 4 Om 5x2 = 40x +75 = 5be— 3-5) x= 3orx=5 ‘The corresponding p-values are y Points of intersection: (3, 4) (5.0) and y «6 yew ae yo-@+2) ~@+2) Bo ax+2=0 @- +2) <0 KEL or x=-2 ‘The comesponding p-values are y Points of intersection: (1, ~3),(~2,0) Poa 3 and y y= xtysdesyednx 6-2 ow-x-2 GIG +1) ‘The corresponding y-values are y = 2 (for x = 2) and y = 5 (forx = —1). Points of intersection: (2,2), (—1,5) 65.2 + y?= Soy? xoysleey 5-=(e-1P =8-2st1 0-28 -2e- 4 = 20+ De 2) xe nlorx ‘The comesponding y-values are y = ~2.and y= Points of intersection: (~1, ~2), (2,1) o. yes yer Pex eox=0 ae + De) =0 x=0,x=-Lorr=1 ‘The corresponding y-values are y = 0, = yak Points of intersection: (0, 0), (— 1,41) @. yes-2tten yecw+3r-1 Bo Weta tar Pot 2e=0 xx- eet) =0 x= 71,02 (-1,-9,,-0,@,1) Section Pl Graphs and Models _9 1 y= VEE y+ REE pal] Points of intersection: (~2,2), (~3, V3) = (—3, 1.732) Analytcaly, — JEFO= =H xt6= BHSr+6=0 (+3) +2)=0 =VF3 (-3.V9) x= -2y=2=9 (-2,2). xt = dx TR. y= —[2x-3] +6 Points of intersection: (3,3), (1,5) Analytically, ~|2x~ 3] +6 = 6 ~ x ao A rE “Gece al) 013 + LL + 207 ‘a graphing utility, you obtain 0.0078 + 4828 + 35.4, = (©) For 2010, t = 60 and =0.13(60%) + 11.1(60) + 207 = 405 acres. (© For 2010,1 = 40 and y = 217, (5.5 Vx)? = (3.29% — 10,0007" 30.25 = 10.8241x? ~ 65,800 + 100,000,000 0 = 1.82413? ~ 65,830.25x + 100,000,000 Use the Quadratic Formula. x= 3133 units ‘The other root, x ~ 2949, doesnot satisfy the equation R = C, “This problem can also be solved by using a graphing utility and finding the intersection of the graphs of C and R. 10__Chapter P__ Preparation for Calculus the diameter is doubled, the resistance is changed by approximately a factor of}. For instance, (20) ~ 26.555 and (40) = 636125. 71. y = (x + Dlx — 4)(x — 6) (other answers possible) (x + $)(x — 2)(x ~ 3) (other answers possible) 7. () y= ke + Smatces 0). 80, (@) If (sis. the graph, then 0 is (~»;) by pans . _ symmetry. Since (~2.)) ion the graph, then oi Un. 7h 7 = HO) + Sok = 2hmy = 2+ 5 Cin) by vans ymmety. Hence te ga 0) y= 2 + kates (0, symimeti with respect fo the origin, The converse is totus For czampley = bas iin symmetry Use (1-9): tuts not symmetti with respect other the as =9 = (IP + kee k= =10,this,y = 4 10, othe yas sip y = (6) Assume that the graph has axis and origin symmetry. Gi 7 — N entchos 1,9) onthe graph, 80 (3, —y) by xaxis Use (1,3: 3 = HIM 9k = 3,thus,y = 38, sey, Se) ko egg, en : Ca 9) by origin symmetry. Gi) ay = kinatches (0 “Thetefore, te praph is symmetie with spect - - tho y-axis. The arguments similar for axis and Use (1,36): (136) = ks k= 36 ths, = 3 : (1,36: (6) thas,3y = 36, See ounce ia enegene wa enema weevil (==, (£0) 85. 2G = OF Fy = 3 = VR OF FO OF 4La? + (y — 3] =F + 9? 4x2 + 4)? — Bay +3632 +9? 3x2 + 3y? — Dy +360 Byte 4 ID=0 Pay sas Circle of radius 2 and center (0,4) Distance from the origin = K x Distance from (2, 0) VEFy = KWAK A Bt yt= KG ar 44+ 99) (1 = K2)x2 + (1 = Ky? + 4K ~ 4K? = 0 Note: ‘This is te equation of scree! Section P.2 Linear Models and Rates of Change 11 Section P.2 10. m 4S. Since the slope i 0, the line is horizontal and its equation isy = 1. Therefore, thre additional points ae (0, 1), (1,1, and G, 0. 17. The equation ofthis line is y-7=-3@-1) yo -3e+ 10. ‘Therefore, three additional points are (0,10), (2,4), and (3, 1). Linear Models and Rates of Change op 4-0 16, Since the slope is undefined, the line is vertical and its ‘equation }. Therefore, three additional points are (~3, 2), (~3, 3), and (~3, 5). 18, The equation of this line is be + 2) yade+2. ye2 Therefore, three additional points are (~3, ~4), (~1,0), and (0,2). 12__Chapter P__ Preparation for Calculus 18.09 Sipe = 2 By the Pythagorean Theorem, 22 = 302 + 10? = 1000 10,/T0 = 31.623 feet. a@ | Bs a eres 885 ay eon lin) (©) The slopes of the line segments are: 29 - 269.7 10-9 a 285.0 — 2823 i= 10) a “Te population increased least apy between 2000 and 2001, aa x + 5y=20 yerbets “Therefore, the slope is m = —} and the y-intercept is (0.4). Box=4 ‘The line is vertical, Therefore, the slope is undefined and ‘there is no y-intercept. 2 yadet3 y= 3x +12 O=3r-4y +12 20. (@) m = 400 indicates thatthe revenues increase by 400 in one day. 100 indicates thatthe revenues increase by 100 in one day. © (©) m= O indicates thatthe revenues do not change from one day tothe next. 22. @) © ~46 -36 ‘The rate changed most rapidly between 20 and 25 seconds. The change is ~4.6 mph/sec. wm o-5y=18 yofens ‘Therefore, the slope ism = § andthe yeep is (0, -3). ‘The line is horizontal. Therefore the slope is m = O and. the intercept is (0, ~1). Section P.2 Linear Models and Rates of Change _13 Be yt 2= 3-3) yH2=3r-9 yeae-l yoaet =o s-0_ 8 ama $a. -8 8 y-0= 86-5) 14__Chapter P 40. m= 0 Preparation for Calculus T= 3/4 _ Ws Wz=0 t +e “athe . Eze 227} dety=-2 arty +2=0 Way #3 = 32x + 40 30x + Ry -37=0 aie stv ste a=3axty3 xty-3=0 Section P.2__Linear Models and Rates of Change _18 55. 2x-y-3 yean3 51. (@) » © "The lines do not ‘The lines appear | +» appear perpendicular “4 "© perpendicular ‘The lines are perpendicular because their slopes | and —1 are negative reciprocals ofeach other, ‘You must use a square seting in order for perpendicular lines to appear perpendicular. Answers depend on calculator used. 58.) © rh ‘The lines donot ‘The lines appear 7 + appear perpendicular. + © perpendicular ‘The lines are perpendicular because their slopes 2 and 3 are negative reciprocals of each other ‘You must use square setting in order for perpendicular lines to appear perpendicular. Answers depend on calculator used. 9. dx —2y=3 Orty yar} m=2 @ yt 2-2) @ — y-2==16+3) yria2e-4 yrdanan3 d-y-3=0 aty+1=0 © -He-2) ) y-2= 1643) ne? yraext3 ° x-yt5=0 16 Chapter P__ Preparation for Calculus 61. Sx~3y=0 yoke @ My + 24x ~53= 0 (63, The given line is vertical @ WyaSay- =2=px-280 (65, The slope is 125. V = 2540 when t= 4. V = 125(0— 4) + 2540 = 1251+ 2040 “The slope is ~2000, V = 20,400 when + V = ~2000(¢ — 4) + 20,400 = ~20000 + 28,400 fea 2 You can us the graphing ulity to determine tha the points of intersection are (0,0) and (2,4). Analytical, Bede 2x — 4 2xlx- 2) =0 = y = 0=> (0,0) xadery= 42d. ‘The slope ofthe line joining (0 0) and (2, 4) is 0)/@ ~ 0) = 2. Hence, an equation Oat 427 yecbed By — 12 = ae + 24 4r—3y +36=0 4. (@) y=0 x= -lartino (6. The slope is 45. V = 156 when 1 = 4 Ve 45(¢— 4) + 156 = 451 + 138 = 245,000 when = 4. (68, The slope is ~5600. V = ~S600(s ~ 4) + 245,000 =36008 + 267,400 mart 3y ator 43 a 4 You can use the graphing vtlty to determine thatthe points of intersection are (0,3) and (3, 0). Analtically, x=0=y=3=9(0,3) =y=0=96,0). ‘The slope of the line joining (0,3) and (3,0) is (0 = 3)/(3 ~ 0) = ~1. Hence, an equation ofthe line is| y-3=-1e-0) nxt3. Section P2__ Linear Models and Rates of Change 17 7. m, im, # me ‘The points are not collinear. "73. Equations of perpendicular bisector: an) Setting the right-hand sides of the wo equations equal and solving for x yields x = 0. Letting x BP + 2) 2e ‘This point lies on the third perpendicular bisector, x = 0. in either equation gives the point of intersection: 74, Equations of medians: 175. Equations of altitudes: (ea) ‘Solving simultaneously, the point of intersection is (ee 1 esipe ote nent ton (8. sm, = UE = BML ~ (C13) _ Ga? — 307 = 2/30) _ 302 ~ 307 1 b= @/3) QB 2be 24 ba + ot =) re see otte iene (5) (0 sng = Wat + BE + Y/D0) ~ (e/3) _ (2302 + 362 + Bet — 202/60) _ 302 = 387 — 2 ° 0- GA) ~ ~b/3 2be ‘Therefore, the points are collinear. 18 Chapter P__ Preparation for Calculus 17 dh ein ie ine oh pois 0,2) ‘and (100, 212). mR! F-32-4C-0) C= MSF — 160) SF -9¢ ~ 160 For F = 72°,C = 222°. 79. (a) W, = 0.75 + 1250 W, = 130x +920 Analytically, 7B C= 034 + 150 Ifx = 137, C = 034(137) + 150 = $196.58, Using a graphing utility, the point of intersection is (6, 17). O75x + 12.50 = 130r + 9:20 33 =05i—x= y= 075(6) + 12.50 = 17. (©) Both jobs pay $17 per hour if 6 units are produced. For someone who can produce more than 6 units per hour, the second offer would pay more. For a worker who produces less than 6 units per hour, the fist offer pays more. ‘80, (a) Depreciation per year: om = si7s 15 ~ 15x where 0S x5 5. i . 81. (a) Two points are (50, 580) and (47, 625). The slope is 2S = 580 _ a 30 p ~ 580 = ~15(x ~ 50) p= ~15x + 750 + 580 = —15x + 1330 nis. 1 ocx = 2h(1330 ~ p) © 2 ip = 6552 = L390 ~ 65) = Suns. 6 tp =95.2~ {sm -95)= nt () y = 875 ~ 175(2) = $525 (© 200 = 875 ~ 175 17x = 675 = 3.86 years 82. (@) y= 1891 + 3.97 (x = quiz score, y = test score) ® 7 (©) Ifx = 17,y = 18.91 + 3.97(17) = 86.4. (@) The slope shows the average increase in exam score for each unit increase in quiz score. (©) The points would shift vertically upward 4 units. The ‘new regression line would have a)-intercept 4 greater than before: y = 22.91 + 397. Section P.2__Linear Models and Rates of Change _19 83, ‘The tangent line is perpendicular tothe line joining the ‘84, The tangent line is perpendicular to the line joining the point (5,12) andthe center (0 0). point (4, ~3) and the center ofthe circle, (1,1). Slope of the line joining (5,12) and (0, 0) is Slope of the line joining (1,1) and (4, ~3) is 2 143-4 x ‘The equation of the tangent line is ys3 ’ 12y + Sx - 169 = 0. 4-334 24=0 (0) + 3(0) ~ 10] _ 10 _ 10 = owed = H@+30)= 10] 7 85, dr + 3y~ 10 = 0-4 = HOO 2 #6, Ar + 3y— 10 = 0d = HOO =F 2a ond WED+C MW) 21. 5 _ SF VPer Vin 7 ort 1a ons g = H+) +1) “ WTO £9. A pinto he inex + y = 18 (01. The dstance fom the point (0,1 w+ y= $= O's (0) + 10) 5] _ is} 4 Vien ve ae a= is (=1, =1). The distance from the point (~1, ~1) to 3x ~ 4y ~ 10 = is 90, A point on the line 3x ~ 4y = go BEN =A = 10) | [344-1019 CaF 5 ), then By + C = is the horizontal line y = ~C/B. The distance to (x,y) is ()|-ee2- bn Ba + If B = 0, then Ax + C = Ois the vertical line x = ~C/A. The distance to (x,y) is Max +¢l_ [ay + By +c A” are (Note that A and B cannot both be zero.) eh —CONTINUED— 20__Chapter P__ Preparation for Calculus 91. —CONTINUED— ‘The slope ofthe line Ax + By + C toAx + By + C= Dis is ~A/B. The equation of the line through (x,y) perpendicular 5 2a-x) yon Ay ~ Ay, = Be - Bs, Bx, ~ Ay, = Br Ay ‘The point of intersection of these two lines is AxtBy=-C => Aix + ABy=-AC o Bx ~ Ay = Bx, — Ayi=> Bx — ABy = BPs ® By, (By adding equations (1) and (2)) Senet eee (4? + By = ~BC — ABx, + Ay, (By adding equations (3) and (4)) mse 7 a+ (Ag A Afi ta “The distance between (x,),) and this point gives us the distance between (x) y,) and the line Ax + By + C= 0. 0 Se) Seo De Apap « [RC lh (oe + BNC + Ax, + By? wea _ yt By tcl Va 92. y = mx + dao me + (-1ly + 4= qn Mt B+ Cl _ | + (=U) +4 OB Tn + (=F zy 93. For simplicity, let the vertices of the rhombus be (0,0), (2, 0), (,¢), and (a + b,c), as shown in the figure, The slopes ofthe diagonals are then Since the sides ofthe rhombus are equal, a? = B® + c?, and we have Foe ‘Therefore, the diagonals are perpendicular ‘94, For simplicity, let the vertices of the quadrilateral be (0,0), (,0), (,c), and (de) 28 shown in the figure. ‘The midpoints of the sides are (G0) (583) C54) m 65) ‘The slope ofthe opposite sides are equal 22 4 2 ‘Therefore, the figure isa paralleogram. ‘95, Consider the figure below in which the four points are collinear. Since the triangles are similar, the result imme- diately follows. a e qmeyn at ¢ bx ay=qmoy Section P.2 Linear Models and Rates of Change _ 24 96. If m, = —1/m then mm, = —1. Let Ly bea ine with slope ms that is perpendicular to L,."Then my = ~ Hence, m, = m => L, and Le ae parallel. Therefore, Ly and L, are also perpendicular. ‘98. False; if m, is positive, then m, = —1/m, is negative. 22__Chapter P__ Preparation for Calculus Section P.3 Functions and Their Graphs 1. (@) Domainof f: 4x5 4 Range of f: 3 Sy $5 Domain of g: -3 $x<3 Rangeofg: 4S y <4 (© SO) = go) forx = =1 (@ fla) = 2forx=1 (©) eG) = Oforx = —1, 1 and2 3. (@) 0) = 20) - 3 &) (3) = 2-3) © f= 2-3 @ fle- 1) = 2-1) -3=2x-5 @ sO =3-0=3 © v3) =3 - (VF (© s-2)=3- 27 @ -=3-@- Re PHIH? 7. (@) f(0) = cos(2(0)) = cos 0 = 1 2. (8) Domain of ff -S < xs 5 Range of f: -4S ys 4 Domainof g: —4x<5 Range of g: —4S y $2 @ f-2) = -2 a) 2 (© SO) = al) for (© fle) = 2forx (©) els) = O for 4. f-2) = J=BFS (0 = VETS (© f-9) = V=5F3 = =F, undefined (@ fle + Ax) = VF FS viet 6 @ gi) = #6-4)=0 © 88) = QE-4)=8-) = -F © #0 = He- 4) = 0 ~ 4c @) gt +4) = (+ 4+ 4-4) =U 4H =P +8 + Loe 8. @) f(a) = sine =0 ‘Sm wA%)- 2 2m) _ gig(22) oF) 3)“ As) =) _ + AP a _ 2 + Ar + Sed)? + (Aa) = ot o, Hace Be) fi) ot aah ap he OP? geese asa 0 wo, L=) aDaseai ay, £2)=10) _ z= ate vEST 1+ VET aor @-2Ve-1 1+ Ve-1 Dv 1+ VED yr? vp, (2-10) Oat D-Day ee 1 1B. his) = VES Domain: x +3 2 0=9[-3,00) Range: (20, 0] 16. H() = cotr Domain: all # key kan integer Range: (~20,o0) 19. fla) = Je + T= x20 md 1-x20 x20 md = xs1 Domain: 0< x <1 2. We) = OD 1 sinx- 440 1 sins +} Domain al # 2+ 2am, 5a i 52 + ann, mimeger _frtie 1 @ f-2) = (-2F +2 =6 © £0) © fu) © se +2) 42-2 P+ 2=3 fe? +2 + = 2st + Bs? +10 (Note: s?+2.> 1 forall s) Domain: (—20, 20) Range: [2, 00) 24__ Chapter P © s@)=-3+ @ fe +) = -@ +1) +1=-5 Domain: (20,20) Range: (~20, )UL1, 20) 29. fla) = 4—x Domain: (—20, <0) Range: (~00, 00) 32. fl) = px? +2 Domain: (—o0, c0) Range: (20,0) 35. g(t) = 2sin-w i Domain: (—20, <0) Range: [~2, 2] Preparation for Caleulus VEFAx SS m0 (ESS @ f(-3) = J=3F4= Vi=1 ©) f(0) = VOFH=2 © f0) = V5F4=3 @ (10) = (10 ~ 5 = 25 Domain: [-4, 20) Range: [0, 00) 31 hla) = VET Domain: [1, 20) Range: [0, 00) Domain: (-20,0)u (0, e2) Range: (20,0) U(0,c0) Range: [-2,2V2) = [-2,283] intercept: (0,2) svintereept: (— V2, 0) o ~Seos3 Domain: (—20,c0) “ Range: [-5, 5] 36. HO) Section P:3__ Functions and Their Graphs 28 31, Teste ens ?=2 = nin ding he fe “4 minutes. The student is stationary forthe following 6 2 Fy he se 2 si/min ring the final 4 minutes. 3. x-y?=OmyatvE ‘y is not a function of x. Some vertical lines intersect ‘the graph twice. 4. isa function of x. Vertical lines intersect the graph at most once. 4B xt y= daeysaVEne _y is not a function of x since there ae two values of y for 45, yPeat—lay=avE=T _y is nota function of x since there are two values of y for 41, y = fle + 5) isa horizontal shift 5 units to the left. Matches d. 50. y 4) isa horizontal shift 4 units to the right, followed by a reflection inthe x-axis. Matches a Matches e, ‘53, (a) The graph is shifted 3 units tothe lef. —CONTINUED— 48, y = fla) — 5 isa vertical shift oy 5 units downward. Matches b. Si. y = flx + 6) + 2 isa horizontal shift to the left 6 units, and a vertical shift upward 2 units. (©) The graph is shifted 1 unit to the right 40, J=4-y=0= y= VFR ‘ys a function of x. Vertical lines intersect the graph at most once. a ety yoayine _y is nota function of x. Some vertical lines intersect the graph twice. Mv tye4oy 1yis a function of x since there is one value of y for each x 46, By = + 4y Oy = ‘ys function of x since there is one value of y for each x ~f(-2) — 2 is areflection in the axis, a reflection inthe x-axis, and a vertical shift downward 2 units. Matches e. 82. y= fle ~ 1) + 3 is ahorizontal shift tothe right 1 unit, and a vertical shift upward 3 units. Matches (©) The graph is shifted 2 units 26 Chapter P__ Preparation for Calculus 53, —CONTINUED— (@ The graph is shifted 4 units downward 54. @) ale) = fle 4) (6) =f) =1 (0) = f(-4) = ~3 ‘Shift fright 4) @ 8) =f) -1 ‘Vertical shift down 1 unit 58. @) y= Vx +2 Vertical shift 2 units upward (©) The graph is stretched (©) The graph is stretched vertically by a factor of 3. vertically by a factor of 5 ©) g@) = fle +2) © ah) =f) +4 Shift fle 2 units Vertical shift upwards 4 units © ate) = 270) © a) = 470) 8(2) = 2f(2) = 2 (2) = 2/0) =3 a(-4) = 2f(-4) = -6 a(-4) = bya) = Section P3__Functions and Their Graphs __27 = VB © flats) = fle = 1) = VE=T © ef) = o( Vz) = (V8 - = 2-1, 20) 57. (@) f(@(1)) = f(0) = 0 58. f(x) = sin x, g(x) = x (@) flg(2)) = fl2) = sin(2z) = 0 (b) flg(1/2)) = flor/2) = sin(m/2) = 1 © sf) = 80) = 0 sAw/A) = shsialn/4) = o(/3/2) = af 3/2) = 22 ©) flea) = fla) = sin(arx) (f) g( AG) = g(sinx) = wsinx 88. J) = #40) = VE 60. f(5) = x2 ~ 1, g(0) = cose (Fea)la) = s6@0)) = Ava) = (Vay= x 220 (Foa)G) = Fl@(2)) = F(c0s x) = cost x — 1 Domain: [0, 0) Domain: (~20,¢) (le) = of) = 86) = YH -f)ls) = a6? ~ 1) = costs? — 1) Domain: (—o0, oo) Domain: (—20, 00) No. Their domains are different. (f+) =(g «f) for x 2 0. Nofes#8ef 61, fo) = 3.96) = 8-1 (7-80) = s060) = fo? = 8G Domain: allx #21 (“fle = al F00) = &| Domain: allx #0 No, fog # ach 62. (Feahe) = WEF) Domain: (~2,20) ‘You can find the domain of ¢ «by determining the intervals where (I + 2x) and x are both postive, or both negative. Tar Domain: (90, ~4} (0,0) 6. (a) (f-8)() = fe) =f) = 4 © (f-8-3) (1) = —2 ©) (g-fA-1) = a A-D) = 4) 5), which is undefined ©) AAe(-1) = 4), which is undefined 28 Chapter P__ Preparation for Calculus 64. (A <1) = Ale) = A(O.:) = 240.6)? = 0.360 (A +1)( represents the area of the circle at time f. 65. FG) = JE=2 Lets) =~ 1, gs) = De and 0) = V3. Then, (fog =MG) = slele ~ 0) = 20 ~ 1) = VIR=D = VEE= I= FO. [Other answers possible} 66. F(x) = ~4sin(t ~ 2) Let la) = ~4s, (2) = sin x and hla) = 1 x, ‘Then, (Fog +A) = fell ~ x) = flsin(t ~ 2) [Other answers possible] 61. fl-3) = (-2)*4 ~ (3) = 8 — 7) =f) Even 69. f(-2) = (2) eos(-3) = —x008x = ~ fla) oda 70. #(=2) = sin’(—x) = sin(—x) sin(=2) = (sin x\(-sin) © sin? Even 71. (@) IfFiseven, then (3, 4) son the graph. 72, (a) Iffis even, ten (~4,9) son the graph. (© Ityis od, then (, ~4 ison te raph. (© Itfis 08, then (~4, 9) is onthe graph 73, Fis even because the graph is symmetric about the y-axis. «is neither even nor odd. ‘his odd because the graph is symmetric about the origin. ‘74, (a) IfFis even, then the graph is symmetric about the y-axis, (®) If is 064, then the graph is symmetric about the origin. Section P.3 Functions and Their Graphs 29 Thatyta0 ‘79. Matches (i). The function is g(2) = ex Since (I, ~2) 80, Matches () The function is f(x) = cx. Since (1, 1/4) satisfies the equation, ¢ = ~2. Thus, g(x) = ~2x% satisfies the equation, ¢ = 1/4, Thus, f(x) = (1/4}x ‘81, Matches (jv). The function is (2) = ¢/, since it must be 82. Matches (ii). The function is h(x) = eV/[x. Since (1. 3) Since (1, 32) satisfies the equation, satisfies the equation, ¢ = 3. Thus, W(x) = 3V/a. 32/% 83. (a) M14) = 16%, M115) = 23° (©) IFH() = T(¢~ 1), then the program would tum on (and off) one hour later. (© If H@) = 70) ~ 1, then the overall temperature would be reduced I degree. £4, (a) Foreach ime, thre comespond a depth d- 0) 4 (©) Domain: 0 s 1s 5 “ Range: 0d < 30 - © ft a yi (0) ACLS) ~ 345 aces /farm 86.) = 2). ) +a) 0 © nlf) 9m(is) + om = 0,00078125x2 + 0,003125x ~ 0.029 30__Chapter P__ Preparation for Calculus 87. #6) = fl + bx ~2] If < O,then f(s) = ~~ (e~2) = ~2e +2 = 201 ~ 2). 160 < x < 2,thenfa) =~ (e-2) = 2. Wer 2 2,then fa) = x + (e~ 2) = 2x 2=2lx- D. Ths, (2(1-x), x<0 fo) = 32 Osx<2. 2-1), x22 88, p(s) = 9 — x + 1 sone zero. p(t) = 2° — has three zeros. Every cubic + polynomial has at least one zero. Given p(x) = Ax? + Bx? + Cx + D, we have Ay p> ~0088x-»—co and prco as x-H00ifA > 0.Furhermore, p->c0 aS 7 xe and p> 00 a8 x—900 if A < O. Since the graph has no breaks, the 4 ‘graph must cross the -axis atleast onetime. 89. fl—2) = dage (—aPMt ++ + = + aya? + ay(—a) Langer Ho ays? + aya] fe) oa 90. f(=2) = aa (=2)9° + ayy g(-a)2 + tala + a agg + dag 92 Ht an + Og =) Bven 91. Let F(x) = fel) where fand gare even. Then FRx) = f(-2)e(-2) = se) = FO). ‘Thus, F(a) is even. Let F(X) = feel) where fand g are odd, Then [S60 a(e)] = s@)8(2) = FG). 92, Let F(x) = fla)g(x) where fis even and g is odd. Then Fa) = s(-ade(-2) = 0[-ab0)] = ~ fete) = — ‘Tis, Fx) is odd. 93. (8) V = x(24 — 2x) = 4312 — 2) © :O 428) 20, Answers will vary. 34_Chapter P__ Preparation for Calculus Review Exercises for Chapter P Ly=2-3 x= 0=9y=20)—3=-3=5 (0-3) printercept y=0=90=21-3—9x=3=9(3,0) einercept 2 y=(e- De-3) x=0=5y= (0 10~3)=3=9(0,3) intercept y= 0=90= (~ e- 3) = 1,39 (1,0),,0) (04), nme Sex =1=9(1,0) intercept 5, Symmetric with respect to y-axis since (ay = (2 + ay =0 sys at tayo intercepts Aaya4 ‘and y = O are both impossible. No intercepts. 6. Symmetric with respect o y-axis since ya ah (a +3 yoxtaatt3, dya-besd 10, 02x + 0.15y = 0.25 De + ISy = 25 Review Exercises for Chapter P__38 1B. y= VS= My Domain: (20,5) 15. y= 42-25 16 y= 8YE=6 Tar 4= 8 a+ 4y = 20 Xnin= Xan = 4 set y= 20 ‘Xmax = 5 ‘Xmax = 40 Ix = B Xsel=1 Xsel=10 oa ‘Ymin = -30 Ymin = -40 aan Yinax=10 mux = 40 cami Yelns Ysel=10 Point: (4,1) 18 rn yertt @+n-2=7 Ons-246 zt No real ston No plas of nerteton ‘Te graphs ofy = x + 1 andy = + 7do not intersect 18. You need factors (+ 2) and (x ~ 2). 20 y= Mutipy by to obtain origin syne. Iupy by to obtain origin sym. AVE Ener yest vend) (1 = M-2P k= ~fandy = “fe eae (©) 0 = k(0)° => any k will do! @ -1=K-1IPSk=laeye ed mt 720" 1-CD t-en-$ 62 Slove = 5" G72) 36 Chapter P__ Preparation for Calculus Mm — -y-0=-Hx-(-3)) -h-2 w.@ y-4= Leer) 16y — 64= 7+ 14 0 = 1x~ 16) +78 {() Slope of line ‘31, The slope is 850. V = —8501 + 12,500. (3) = ~850(3) + 12,500 = $9950 = 6 = Oe - (-2)) y = 6 Horizontal line as 28, mis undefined, Line so@ y-3=-3e-0) By-9 = 242 a+ y- 11-0 (© Slope of perpendicular lines 1. y-3=1e-1) yore Onx-y+2 © yo3=1G-) yoxt2 O=x-yt2 @ yn3 y-3=0 251 + 13.501 + 36,500 22.751 + 36,500 (b) R= 30 (© 301 = 22.751 + 36,500 71251 = 36,500 1 - $034.48 hours to break even 32. @) Review Exercises for Chapter P__ 37 Bx-y=0 yas [Not a function of x since there are two values of y for Me-yno Beye 8 aos ony Fencono snc tei Fonction fe one Nea fnsion tse wae ery ren fawoty trent thc we twos toms mst=2 Me) Ho) C4 42-18 Qeeme -4 <0) © f= [0-212 (a) £0) does not exist. f= | © fl) = [1-2 =1 Meads) 38, () Domain; 36 = 2 0-9-6 5 x56 or [6,6] Range: [0,6] (b) Domain: allx #5 or (—00,5)U(5, 00) Range: ally #0 or (—00,0)U (0,00) © Domain: allx or (—o0, 00) Range: ally or (—c0, 00) 40, (2) = 1 = and g(a) = 20 +1 (@) fx) ~ gz) = (1 ~ 93) — Qe + 1) = mx? = Oe ©) fO)ela) = (1 — Qe + 1) = —28 PH BEET © els) = a(t ~ #) 21 =) #1 = 3-28 38 Chapter P Preparation for Calculus (@) The graph of gis obtained from fby a vertical shift ‘down I unit, followed by a reflection in the a-axis a0) = Le) - 1) P+3e+ 1 43, (a) Odd powers: f(x) = x, g(x) = 2°, ha) = 2 y ‘The graphs of fg, and h all rise to the right and fall the le. As the degree increases, the graph rises and falls more steeply. All three graphs pass through the poiats (0,0), (1,1), and (—1, =1). (©) y= 2 will look like MG) = _y = 38 will Look like h(x) = 44, (@) fla) = 20 - OF (b) ax) = (©) fa) = 0 bat rise even more steeply. ee OF (©) The graph of gis obtained from fby a vertical shift upwards of I anda horizontal shift of 2to the right. ‘The paps off, and al rise to the lef and to the right. AS the degre increases, the graph rises more ‘steeply. All three graphs pass through the points (0, 0), (1,1), and (—1, 1). but rise and fall even more steeply. N 45. @) 2 (12 ~ 2) Problem Solving for Chapter P__39 (&) Domain: 0 < x < 12 (© Maximum area is A = 36, n general, the maximum area is attained when the rectangle is a square. In this case, x = 6, 46, For company (a) the profit rose rapidly for the first year, nd then leveled off. For the second company (b), the profit dropped, and then rose again later. 47. (a) 3 (cubic), negative leading coefficient (©) 4 (quartic), postive leading coefficient 48. @) © 2 = 1.204" + 64.2667 (©) The data point (27,44) is probably an error. Without this point, the new model is y= ~ LAB + 66-4387. Problem Solving for Chapter P L@ 6x+y?-8y=0 6x +9) +(2-8y +1) 9 +16 @— 3h + (y— 4 = 25 Center: (3,4) Radius: 5 (© Slope of line from (6,0) to (3,4 is Ske ote ae 3, He, (© 2 (quadratic), negative leading coefficient @ 5, postive leading coefficient 49. (a) Yes, ys a function oft Ateach times, there is one and only one displacement y. (&) The amplitude is approximately (0.25 ~ (~0.25)/2 = 0.25. ‘The period is approximately 1.1. (© One model is o¢ ue (©) Slope of tne fom (0,0) t0 (3,4 is & lope of tangent tne 5-3. Hence, y-0= 3-0) @ 3 4 3 2 40__Chapter P__ Preparation for Calculus 2, Lety = mx + 1 be a tangent linet the circle from te point (0, 1). Then B+O+Pet + (me tie Ded (oP + I? + Ame +3 =0 Setting the discriminant &? ~ ac equal t0 ze, 16m? — Ain? + 19) = 0 16m ~ 12m? = 12 2 aS ‘Tangent ines: y = V3e + Land y = Vix +1 (@) HQ) ~2 i; ©) He -2) @ H-x) © ~HG) © mG) (6) -H@e—2) +2 4. (@) fe +1) i © 26) © ~f0) i @ AD i 5. (@) x+2y= 100 => y= A) = 9 =f Domain: 0 < x < 100 © Maximum of 1250 m? atx = SOm,y = 25m. Problem Solving for Chapter P__41 © Fo) +1 : @ fC) ; ¢f) [09] i (© AG) = —}oe4 ~ 1005) ‘a2 ~ 1o0e + 2500) + 1250 Hox ~ 507 + 1250 A(50) = 1250 mis the maximum, x= 50m,y = 25m 42_Chapter P__ Preparation for Calculus 300 ~ 3 6 (@) 4y + 3x = 300 = y= Als) = 203) = (2023) - =e a0 Domain: 0 < x < 100 © 4 Maximum of 3750 8 at x = 50 ft,y = 37.5 ft. 7. The length ofthe trip in the water is / 77 37, and the length of the trip overland is JT + (3 = 3 Hence, the total time is Je +e, 3) ) Slope rt 4 ‘Slope of tangent line is less than 5. ‘= 3. Slope of tangent line is greater than 3. (© AG) = -3o ~ 1005) 3 (4? — 100x + 2500) + 3750 (x = $0)? + 3750 A(S0) = 3750 square fet is the maximum area, where x = 50 ftand y = 37.5 ft | Letd be the distance from the starting point tothe beach. distance Average speed = SESE (©) Slope = 7" = 4.1. Slope of tangent Kneis less than 4.1 a+n-s0 0 seme SE _@thn4 h thee * eat hheO (©) Leting h get closer and closer to 0, the slope approaches 4. Hence, the slope at (2, 4) is 4 (@) Slope = 5 322 supecrunatin i petra - t oy sope «2=1 spe tpn ts a2 _10 far 47 at £14 + h) = f(4) Gean-4 ATH iG OFA = 2 JUFH- 2 JETR 42 a barns? _Wsn=4 Warn) (© Soe = Slope tage is eter tan @ Slope = © 1 “za Tar iee [Ash gets closer 0, the slope gets close to 4. The slope is atthe point (4,2. mo BEF we 6x + B+ 6r-9e0 6s VOTH 2 =e <3 + VIB ~ 1.2826, -72426 ~-— Fey aye Ge —3P +9 = Ae + 7) Ge +94 y= DF + 29? it ytt6r-9=0 + 3P+ yw Is Circle of radius YTB and center (—3, 0). o Problem Solving for Chapter P 43 44__Chapter P__ Preparation for Calculus () k= 3,4 2% +7 = 12 1 DO REF ee Ge ~ 4 + yPaaGe +99) (ke — Ix? + 8x + (k Dy? = 16 (© As k becomes very large, - 4 ‘The center of the circle gets closer to (0, 0), and its rads approaches O. 1B 4dy=1 [a+ OF + Me PD G+ Pe — DP + Ale +P + IP tt (2-1 + ya + a+ y= 1 xa De 1+ Dey + Ot y= Got + 2329? + yf) — 20? + 297 = 0 (2 +3 = 262-9) 28 = x=0 or = Lety = 0.Then x ‘Thus, (0,0), (V2.0) and (~ V/7,0) ae on the curve. (@ Domain: allx #1 Range: ally #0 Domain: allx ¥ 0,1 (©) The graph isnot alin. It has holes at (0,0) and (1, 1. CHAPTER 1 Limits and Their Properties A Preview of Calculus . . . Finding Limits Graphically and Numerically»... . . a7 Evaluating Limits Analytically .......+~ 37 Continuity and One-Sided Limits... 2... 6.6 - 8 CHAPTER 1 Limits and Their Properties Section 1.1 A Preview of Calculus 1, Precalculus: (20 ft/sec)(15 seconds) = 300 feet 3. Calculus required: slope of tangent line atx = 2 is rate of ‘change, and equals about 0.16. 5, Precalculus: Area = }bh = 4(5)(3) =f sq, units 1F + — SP = V6 + 16 ~ 5.66 at aaa an (©) Inorese the number of line segments. Graphically and Numerically Finding Lit Section 1.2 Vly 19 1.99 1.999 | 2.001 | 2.01 21 Fle) | 0.3448 | 0.3344] 0.3334 | 0.3332 | 0.3322 | 0.3226 03383 (Acta limit aly 19 1.99 1,999 | 2.001 | 2.01 21 fle) | 0.2564 | 0.2506 | 0.2501 | 0.2499 | 0.2494 | 0.2439 tin $2 ~ 025 (Actua timtis}) me 4 * ayy 01 0.01 0.001 | 0.001 | 0.01 OL fe) | 0.2911} 0.2889 0.2887 | 0.2887 | 0.2884 | 0.2863 (acta imi is (V3) fs — = 02887 48 Chapter 1__ Limits and Their Properties f 31 3, x [- =301_[-3001 | -2999 | -299 | -29 76) | —0248s [ -02498 | -02s00 | —02500 | ~02s02 | —o2s16 tin, AA=F=2 995 (Acuatimitis 5 x | 29 | 299 | 299 | sor | 301 | 31 76) | —006e1 | —00807 | —o062s | —o002s | -o0623 | —o.0610 tg WAG + 1 1/4) 9.0505 (Acta iit is -%) ele 39 3.9 3.999 | 4001 | 4.01 41 ft) | 0.0408 | 0.0401 | 0.0400 | 0.0400 | 0.0399 | 0.0392 tig BAILS) 904 (Acta titi) 7x ~O.1 0.01 0.001 | 0.001 | 0.01 OL 76) [0.9983 [099998 [1.0000 | 1.0000 | 0.99998 [0.9985 tg $ 0000 Act im is 1 Make sue you we radian mode) Lo 0.1 0.01 04 * [-a [- =oo01 | oo | oor | oa 70) | 0.0500 [ a00s0 | 0.0005 | -o.000s | - 0.0050 | —0.0s00 tio! oom00 Ac it :0) Mat sue you we alan moe) 9. lim (4 — 2) 1 10. lim (<7 + 2) 3 AL, tim f(s) = fim (4 ~ x) = 2 18, ti = $l doesnot exist For values of 10 the lettof 5, [x ~ 5|/(e ~ 5) equals ~1, whereas for values of xt the right of 5, [x ~ 5|/(e ~ 5) equals 1 15, fim sin wx 17. lity cos(1/x) does not exist since the function oscillates between ~1 and I as.x approaches 0. 12, im fla) = lim G2 +2) = 3 14 iy tes exis set fnton nee ad ee dot ds x apts 3 16. tm see x 18, lim, tan.xdoes not exist since the function increases and th decreases without bound as x approaches 2 Section 1.2 19. (@) fl) exists. The black dot at (1,2) indicates that f= 2 (©) lim f(0) doesnot exist Asx approaches 1 from the Tet, f(x) approaches 2.5, whereas asx approaches 1 from the right, f(x) approaches 1. 20, (a) s(-2) does not exist. The vertical dotted line indicates that fis not defined at 2. (©) lim, (2) does not exist. As x approaches ~2, the ‘Values of f(x) do not approach a specific number (© f(0) exists. The black dot at (0, 4) indicates that 70) © im J) doesnot exist. As x approaches 0 from the {eft 2) approaches 4, whereas asx approaches 0 from the right, f(x) approaches 4, 21. im f() exists for llc # ~3.Inpatcua, tim Bo Finding Limits Graphically and Numerically 49 (© ld does not exist. The hollow circle at (4,2) indicates that f is not defined at 4 (© lim fa) exists. As x approaches 4, fs) approaches 2: fim fle) = 2. (© 112) does not exist. Te hotow ctl at (29 ‘indicates that f(2) is not defined. (0 im fs exits As x approaches 2,0) approaches Jims) = (@ 8) exits. The black dot at (4,2) indicts hat (4) = 2. (h) tim f(x) does not exist. As x approaches 4, the values GFF) do not approach specie mimber. 22, lim fl) exists forall ~2, 0. In particular, im, fa) = 2 © [ss [4 [3s [36 [37 [4 [15 | 225 [225 | 22s [225 | 225 | 225 lim, c@) = 225 © fa [as [29 [a [1 [35 [4 [12s | 19s [1s [ 19s [ 22s | 22s | 225 ly C() doesnot exist The values of € jump fom 1.75 102.25 at = 3. $0 Chapter 1__Limits and Their Properties 28. CU) = 035 ~ 0.12f-(¢- Ol: [3 [33] 34| 35 [36] 37] 4 co | 059 on [on [on fon [07 [o7 lim, C() = 0.71 ss ©l [2 [as]29] 3 [aa] as] 4 et [047] 059| 059 | 059 | 07 [071 [or lim CU) does not exist The values of C jump from 0.59 10 0.71 at ¢ = 29, We need [/(2) ~ 3] = [Cx + 1) ~ 3] be- 2] = [e+ -3) fx — 2] < 04, Hence, take 5 = 0.4. 1F0 < 2] £04, then Ise) ~ 3] < 0.4 as desired 30, We need [f(x) ~ 1] [y= f= Bea 0: la+3)-s]<0 bk- 0 l@-1)- Cal ce [kt 0: B-3) 0 wll work Hence for any 8 > 0, you have B-3l 0: Ier+5)- Cd] 0 [Ge+9)-a] 0: |-1—(-D] <@ Oce Hence, any 5 > 0 wil work, Hence, for any 8 > 0, you have I-D-Cil 0: [YF-ol 0: Ik-a-a ce [-G-2)-4] <2 G-2<0) Jex- a= [+2] =e COl 0: [@+1)-2I <6 pP-i| 0: |ve-a 0: lk-3]-Ol 0: Ite #39) -O] 0 except x = 9. The graphing utility doesnot show the hole a (9,6). 53, fm (0) = 25 means thatthe values of approach 25 asx gets closer and closer to 8. ‘56, (D) The values of f approach different ‘numbers as x approaches ¢ from diferent sides of 57. (0) C= Dar rake ha2~asseen 6) 4e=$5,r=$8~ 087850 We = 65,r= SS ~ Last em ‘Thus 0.87535 0, there exists 6, > O-and 8, > O such that = el < & = [/la) ~ Lal < e:Let 8 equal the smaller of 8, and 6,- ‘Thea for fx — e| < 8, we have [Ly ~ Lal = [Ly ~ 4) +46) — Lal s [ty ~ FO) + [f0)- bo] < ete. ‘Therefore, |Z ~ Lal < 2e. Since e > 0 is arbitrary, it follows that L, 70. $0) 10 = me + byni # O.Let 6 > Obs given Take 8 = 77. Oi 10? = 1 < Oand 90x? - 1 < 0, -(- wala) = (55-759) For all x # 0 in (a, 6), the graph is positive. You can verity thi witha graphing uty. 73, Answers will vary. 74, my ly ‘71, im [/(0) ~ £] = O means that for every « > O there exists 8 > 0 such that if Oek-d 0. Lete = HL. There exists 8 > O such that 0 < |x ~ ¢| < 6 implies that le) — 1] <= E Thatis, ego) -L<$ $< sa) < For x in the interval (¢ ~ 8,¢ + 6), x # c, we have ate) > E > 0, ws desire, a ster n |4+ oir] 4+ [o1y)|| » [4-lorr| #4 —[o.1)) 1| at 7 1[ 39 69 2| 401 701 2| 399 699 3 | 4001 zoo |[3 | 3999 | 6999 4 {| ao00r | 70001 |[4 | 39999 | 69999 ‘75. The radius OP has a length equal to the altitude z of the fs h ait triangle plus 5. Thus, z= 1 ~ 5 ving =i Aaneimge = ses toe mec. - "76, Consider a cross section ofthe cone, where EF is a diagonal ofthe inscribed cube. AD = 3, BC = 2. Let x be the length ofa side ofthe cube, Then EF = xJ/2. By slr ingles, EE _AG BC” AD wi 2 3 Solving for x, 3Vie= 6 — (v2+2n=6 Section 1.3 7 (@) lim He) = 0 4 { je © im, hs) = 6 @ limsta) = 0 © lim, fa) ~ 0524 1 (-4) ) = x008 x Evaluating Limits Analytically Bw Section 1.3 $0) = te - 4] Evaluating Limits Analytically (@) lim e@) = 24 © lim s) = 4 @) tins = 0 © Jim 0 = 7 $8__Chapter 1__Limits and Their Properties, 5. lim = 2*= 16 6. Jim, 8 = (-2° 7. ty (2x — 1) = 2(0) = 1 = =1 8 lim, (x +2) = 3-3) +2 3 + 3{-3) 9. Jim, G2 + 33) = =o 10. tim (-? + 1) =P +1 =0 M1 fim, Qe + 4x + 1) = 2-37 + 4(-3) +1 =~ 12+ 7 12, lim Gx ~ 22 + 4) = 31) - 20)? +4 = 5 14 tim, 25 = dmyrya "3a 77? 16. tim 2&3 10 tim sSE— = SO 35 38 "extsS 345 8 Ca Vae2 VT+2 VO 3 18. tim 221. SFT 19. tim JF I = JET =2 20, lim Ye+4 = YaFS = 2 21. lim, («+ 37 = - 22, im (2x ~ 19 = [2(0) ~ 1P = ~1 23. @ limfG) =5~1=4 (©) lim eG) = 4 = 64 (©) lim (7G) = e(F(1)) = (8) = 64 24, (a) im, f) = (-3) += 4 (©) lim eG) = 4 = 16 © im, 70) = (4) = 16 26. (a) tim fe) = 24) — 3(4) +1 = 21 (b) lim g(x) = YU F6=3 (©) im o( 6) = (21 ign tan x = tan = im cos ™ = cos22 = 1 28. im tanx = ton = 0 29. tim cos2Z = -} i. lim sec 2x = see py OE = 08S 37. (a) lim [S90] = 5 fim (0) = 5) = 15 (©) im (70) + 2] = tm la) + tin a) = 2+ 3 = 5 (© Hina (oe) = [in in 63] = (2918) = 6 © ty = AD EG) ~ tim @G) “3 39. (Lim L/P = [lim fi = (4) = 64 (©) tim 7G) = flim 7G)= V4 (©) lim (3/6) = 3 tim FG) = 3(4) = 12 =2 © lin [/09F* = [in Fp = (= 8 2x + 1 and (x) = =22** at. fle) x=0. (@) fim e(4) = im f(2) = 1 agree except at (©) im, 92) = im, 70) = 3 & agree except at 48, /6) = abe + 1) and ela) == (@) im (0) = tim 70) = 2 aba) = Jim, fla) = 0 1 agree except atx = ~1. Section 1.3 Evaluating Limits Analytically _ 59 38. () tim [409] = 4tim fa) = 4(3) = 6 (© iy 0) + 0 = i) + im 9) = 3 45 = 2 mst ayn) _3 ein ()(3) <3 BESO) _ 3/2 Tim g@) "1/2 © lin 700 in £2) © line) : 4, 0) lin 70) = RFR) = YA = 3 tte) _ Sims) _ 27 3. © in ig = gis 7182 © tim [/O)F = [lim #6) = (27)? = 729 @ lim LAG)P? = [lim Flo? = O77? = 9 42, #0) = x 3and Ma) = == ogre except ax = 0. @ Ma) = im, 7) = (b) Him A(x) = tim F) = 4. g(2) and f(s) = = (@) im f(x) does not exist. (©) tim f) and g(x) = 2x — 3 agree except at 60 Chapter 1__Limits and Their Properties ar. 9) = 2=8 = ims) = in) = and g(x) = 22 + 2x + 4 agree except at s in 335 = ln HGH + 3)@-2) AGF3G=3) 8, tin EES HSS = tpg VETS = VS, VETS + VS tS ay Vers V5 +5 “I Vees+ 5 4, jig PE E= Ay VETS VE ET La Vitat+ 2 etm 2t#=2 ea) Je¥3 3 JES +3 Jer 543 _LezT +2 1 yet A im» “Marea” or3 6 rm oa Sete Boaters Bia a 1 “IR oF a BIO =a? ~ x L agree except at Bim fl) = Jim, g60) = 3 80 Ine 4 "RG e+) (= 9-1) ERQ= aes 2) = tim f= * Bas) 31 ala =3 Section 1.3 Evaluating Limits Anatytically 64 Go, og SPARS yy St AE EP yp WOE mos a) = 28 (et Ba = 2b + Aa) += GP = 24 1) _ gg, At Balke + (Aa)? — 2x — De + A = te = ‘ar ay a 61. Jim, = Jim, Qx + Ar -2) = 2¢ 2 tim SAN My Sate + Sas)? + (Be — 2? am Ar a ae py Ae + Sud + (A) fim, Ge? + ade + (0) ain, ae 68, jig FFE~ VE. os x 01 0.01 | -0.001 | 0 | 0.001 | 0.01 | 01 ° ye) | 0358 | 0354] 0254] 7 | 0354 | 053 | 0349 + i Veva- Ji _,, VeF3- Ji YETI + VI Analytially, tim YF 2— V2 jpg YE 2— V7 EFI + VE yea a e Jett JB iy AZ = 2 tg ts 1 2 0354, Jer 2+ J) 0 Je+2* 2 22 4 x [iso [1599 | 15999 [16 16001 [i601 [16 F(x) | - 0.1252 0.125 | 2 | -0.125 | —0.125 | —0.1248 7 4 ‘Analytical, im, ina Gey iwically, Ry x16 ~ 18 (Ja + a va — 4) x [-o1 | -oo1 | -0001[o] oon] oor | o1 ‘F@) | ~0263 | —0.251 | -0250 | 7 | -0.250 | 0249 | -0238 14 Tee 2 Anaya, ig — = Ht +a) 1 a = x7 Emery x MGs 6 _Chapter 1__Limits and Their Properties = U x | 19 | 199] 1999] 199% [20] 20001] 200] 20] 21 7 . Fe) | 72.39 | 920 | 992 | 999 |? | 8001 | e008 | soso | sear = may, 222 = yy = 2 2 2 + = lim (x4 + 20° + 4? + Br + 16) = 80. (Hint: Use long division to factor x5 ~ 32.) 1=2282)]- Gyo) =0 = (0) =0 7A, Jim dee d= w(— =) =0 75. tim, 28% — tim sine = 1 Sth cotx Othere exists @ lim f() exists, so would tim [f(x) + g(2)], which is a 8 > O such that [f(3) — b| < ewhenever |x ~ el < 5. Contradiction. Hence, lim g(3) does not exist. Since [f(2) ~ b| = [b ~ b| = 0 < eforany & > 0, oe then any value of 8 > 0 will wrk. 108. Given f(s) = x, i a positive integer, hen 108. 175 = 0, then the propery is tre because both sides . = ane equal 00. If # 0, let e > O'be given, Since Bae tn eed) fife) = rote ese 8 > Osean im atin *] = [him (x9) [7l) ~ L| < e/|b| whenever 0 < |x — el < 6 Hence, {im in 2] = elite) wherever 0 < [x ~ | < 8, we have WIL) — 2] <2 or |bfts) — bl < © which implies that Lim [6/¢8)) = bb. 110. Given jm fa) = 0 ML =MIs(o| < feet) s mIFC | For every e > 0, there exists 8 > Osoch that Jim (aA 00) him fala) im Cl) [fle ~ 0] < evwheneverO < [sel < 8 ane er a Ss amen ace -M(O) < im feta) = (0) |r ef < 8 Therefor, lim [(] = 0 05 limstaats) <0 ‘Therefore, lim f(x)e(x) = 0. 112, (2) Im [/(0)] = 0, hen fim [0] = 0 (©) Given lim (a) = L: = [p60] = 6) = [709] Tarren @> Ost > On Hig U1) $ ti ste) Yim | ee el < WS We) tl <2 0 lim) < 0 [r= el <8, then Lim [70] = [2 Therefore, lim fa) = 0 113, False, As x approaches Offom the 114, False. li 115, Tue ven, B= 1 116, False, Let 117, False. The limit doesnot exist 118, False, Let x ret “ () = Be and g(a) = 3, po (224 = SG) = Band gf) ae ro ‘Then f(x) < g(x) for all x # 0. ‘Then li fa) = 1 but (0) #1. But im fl) = tim g() = 0. Section 1.3 Evaluating Limits Analytically 617 uote sa jig SRE yg ens, Le 4 ttez0 : sits won| fee iy eons)” ESGT + coos} ig | = tia = sax _ sax EBay T+ cose lim) does not exist since forx < 0,f(3) = Feora~4 -[s[s Fal = (0) = 0 sau) () cea wa = 00 testo © Ji fC) does not exist. [No matter how “close to” 0. is, there are still an infinite ‘number of rational and irrational numbers so that lim f(x) does not exist. we toale Jim e() = 0 When xis, “elose to” ose to" 0, both parts ofthe function are sextl sex tT La co _ jpg La eose 1 Hoos Lees 1 0 Ia TF os om 2 = tg Dot : cooxm 1-48 ex = BREW cos) - ames © cos(0.1) = 1 = $(0:1)* = 0995 (@) c0s(0.1) ~ 0.9950, which agrees with part (¢). 124, The calculator was set in degree mode, instead of radian mode. 68 _Chapter 1__Limits and Their Properties Section 1.4 Continuity and One-Sided Limits 4. @) lin se) =1 2 tim, 0) = -2 3.0) Jims) = 0 ©) Jim ft) = 1 ©) lim. fa) = -2 (©) Jim 70) = 0 © lig fl) = 1 © im, s0) = -2 © lim se) = 0 Tae fineton i continous at “The function i continuous at ‘The function is NOT continuous at x = x=3 4 @ im, fa) = 2 5 @ lim fo) =2 6 @ lim, sf) =0 © im fla) = ~2 ©) tim. fa) = 2 (© lim fx) does not exist ‘The function is NOT continuous atr=4. "The function is NOT continuous at © im, f09) doesnot exis. a-x at & Igor a” Ma 4 =2, Jira 7 aea aa4 “Rao 9. does not exist because 10. im —* AR. Fe=5 ‘decreases without bound as x» 3°. ~ GS) at “xe +0) 14 jim HARE e+ As) GP ta)_ 4 + 2aAs) + (Oa) et Ave ali. a ts & tm 2Ma) + (An? + Ae - ax lim. @e+ ae +) de+O41=2r41 15. tim f(x) = jim. Section 14 Continuity and One-Sided Limits 69 16, in. fa) = fig (-s? + 4-2) =2 17, in. 0) = Jig + 1) = 2 ig 10) = i (2 — 4e +6) = ‘in 0) = Ji (8 +1) = 2 Eigse) =2 sim) = 2 18 im fl) = Jin, (13) = 0 1. lim cot does not ens 20 im, 0x des not exis since 21. Jim. Bb ~ 5) = 318) ~ Gi = 3f0r3 sx <4) Lim, cotx and lim cot x do not Jim, secxand lim, sec xdo ay i ate AD exist. not exist. 22, lim, (2x ~ bx) = 2) -2=2 23, lin (2 ~ [AD does not exist because lig @ - Ia) = 2-3) and lim @-[-x) =2-(-) = 6 - -H 28. fa)= 34g 26. fo) == a py = has discontinuities atx = ~2 and has a discontinuity atx = has discontinuities at each imteger Since f(-2) and f() ae not since f(~1) isnt defined. Aesince lig) li J) defined xe = 1 has adiscontnity atx = 1 since (1) = 2 lim 70) = xe 29, fe) = JB= Fis continuous 30. f() = 3 VT=Fiscontinwows 31. ip f(s) on{-5.5], on[-3,3]. eta 32, (2) isnot defined. gis continuous 33, f(s) =a? — 2x + Lis cominvous 34 f(x) = = is coninvous forall on{-1,2). for al real x. reals, 35. f(2) = 3x — cos xis continuous forall real x. 36. flx) = cos is continuous for all eal x. 3 fe) =a for x # 0,x = 0 is aremovable discontinuity, whereas not continuous atx = 0,1. Since = is anonremovable discontinuity. Fel 38. fl) = 57 has nonremovable discontinuities atx 39. f00) is continuous forall real x. and.x = —1 since lim f(3) and im, f(x) do not exist. 10 Chapter 1__Limits and Their Properties 40.56) = 53 has @ nonremovable discontinuity t x = ~3 since lim, f(x) does not exist, and has a removable SiScontinuty atx = 3 since igs = tin 5 = 5 42. f(9) = has a nonremovable discontinuity at x= ~2 since sim, 0) does not exist and has a removable Giscomiuity atx = 1 since i 0) = in 5 = DGD) Lox 10 -CF IG has a nonremovable discontinuity atx = 5 sine lm f(a) doesnt exist and has 8 eorabledlscontiy a y= ~2since Bas = Bn = 43. fo) = EAD x2 tas a nonremovable discontinuity atx = ~2 since lim, fla) does ot exis. 44, 0) = £4 nas a nonremovable discontinuity atx = 3 since lin 0) doesnot exist. ee son{e iE) tas ponte cotta = 1 1 f(l)=1 Ji. I= B= romeo! 3. £01) = tim f) Lis continuous atx = 1, therefore, fis continvous for all weal x E+1 x52 Box x>2 thas a possible discontinuity t x = 2. 4 fQ)=F41=2 im f(x) does not exist. sae elie) Him, f(s) = Jim, (3 ~ 2) = 1 ‘Therefor, fhas a nonremovable dsconinty at x= 2. 243, <1 asf has a postble discontinuity at x= 1. Ls=P=1 2, ip S@) = li (—28 + 3) = "Yin =1 Hig fs) = igs? = i 3. 0) = lim 0) Fis continuous at x = 1, therefore, fs continuous for all eas. x<2 x>2 48. fs) bas a possible discontinuity at x = 2, 1, f() = -202) = -4 2, ali. FO) = isp. 28) = ~4 ]ugecon “Tig #0) ="im (2 — 4x +1) = —3) not exist ‘Therefore, fhas @ nontemovable discontinuity at x22. fan, be 2 Isxss x5 ‘has possible discontinuities at x = 1,x = 5. Sz Buz Mo) = oe eG 2: im 6) = 2 Jim f) = 2 3.41) = lim so) (5) = ti) {18 continuous at x = 1 and x = 5, therefore, fis ‘continuous fr all real x. 1. fll) = ose 52, f(s) = tan has nonremovable discontinuities a ach 2k + 1, kis an integer. ‘54, f(s) = 3 ~ [a] has nonremovable discontinuities at each integer k. 56, fim f(s) = 0 lim fl) = 0 Fis pot continuous atx = v] I 4sinx _ 4 Jip c= Sp i, £6) = fi, ~ 29) Leta=4, 59, Find a and b such that lim, (ax + 6) = ~a + 6 = 2and Jim (ax + 6) = 3a +b = ~2. Sh) = 0. tin 66) = Nin a = lim (e+ @) = 2a Find a such that 2a = 8 => a= 4. xs-l -lex<3 x23 61. f(g) = & = 1 ‘Continuous for all eal. Chapter 1__Limits and Their Properties 2. figs) = Nonremovable discontinuity atx = 1. Continuous forall xl 64. lel) = sin? ‘Continuous for al real x 65. y=[)-x [Nonremovable discontinuity at each integer os BOR 153 Nonremovable discontinuity at x = 3 69. sa) = eT Continuous on (~20, 00) 1. 16) = seo ‘Continuous on: 6, ~2),(—2,2),2, 6), (6, 10),. .« 73. f0) = S* “The graph appears to be continuous on the interval [4,4] Since /(0 is not defined, we know that fhas 1 discontinuity atx = 0. Tis discontinuity i removable 0 it doesnot show up onthe graph, 6.60) = GE HTe Nonremovable discontinuities at x = 1 1 6 = FED Nonremovable discontinuities at x = ~1 and x = cose 1, <0 : w10-[ eo F(0) = 5(0) = 0 7 7 ig 0) = jig 28E=D > ‘ig, £0) = Jim, (5x) = 0 £(0) and fis continuous on the (was the only possible discontinuity) 10. fla) = Ve Continuous on [~3, 20) xt 72. #6) =* Fe Continuous on (0, 20) 14.6) 24 ‘The graph appears to be continuous on the interval [4,4] Since /(2) is not defined, we know that fas discontinuity at x = 2. This discontinuity is removable s0 it does not show up on the graph. 75. f(a) = xt ~ 2° + 3s continuous on [1,2} ‘£(1) = Hand f(2) = ~4 By the Intermediate Value ‘Theorem, f(c) = 0 for atleast one value ofc between Vand 2. Th. f(s) = 32 ~ 2 ~ cos xis continuous on [0, 2 1(0) = ~3 and f(n) = 22 — 1 > 0. By the Intermediate ‘Value Theorem, f(c) = 0 forthe least one value of ¢ between 0 and 7. 1. fG)e8+x-1 FG) is continuous on {0,1}. 0) = ~1 and f(t) = 1 By the Intermediate Value Theorem, f(x) = 0 for atleast one value of c between O and 1. Using a graphing utility, wwe find that x = 0.6823. BL. g() = 2eost ~ 3¢ «is continuous on [0, 1) (0) = 2 > Oand g(1) ~ 19 <0. By the Intermediate Value Theorem, g() = 0 fora least ‘one value c between O and 1. Using a graphing utility, we find that 1 ~ 0.5636, 83. fa) =x +x-1 {8 continuous on [0,5]. (0) = ~1 and f(5) = 29 -1<<2 ‘The Intermediate Value Theorem applies. stxcieil Bexod (e+ Ale 3) =0 x=-4or 4s not in the interval.) Section 14 Continuity and One-Sided Limits 73 16. f(s) = x + 3x ~ 2s continuous on (0, 1} S(0) = ~2 and f(t) = 2 By the Intermediate Value Theorem, f(x ‘one value of ¢ between 0 and 1. = O for at least tan 7 js continuous on [1,3] 78 ft x 8 $l) = 4+ un < and 7) = ~$ + an 2 By the Intermediate Value Theorem, f(1) = 0 for atleast ‘one value ofc between I and 3. 80. fx) = 8 + ae 2 {£68 is continuous on (0, 1}, (0) = ~2 and f(t) = 2 By the Intermediate Value Theorem, f(x) = 0 for atleast ‘one value of ¢ between O and 1. Using a graphing uility, we find that x = 0.5961. 82, M0) = 1+ 6- 34nd ‘his continuous on [0 1 Ho) = 1 > Oand Hl) ~ ~2.67 < 0. By the Intermediate Value Theorem, h(@) = Ofor atleast ‘one value @ between O and 1. Using a graphing uility, we find that 6 ~ 0.4503. 84, fla) =F - Gr +8 {Fis continuous on [0,3] f(0) = 8 and f(3) = —1 -1<0<8 ‘The Intermediate Value Theorem applies. Fo 6r+8=0 2-4-0 orx=4 ¢ = 2c = 4's not inthe interval) ‘Thus, f(2) = 0. 14_Chapter 1__Limits and Their Properties 85. fla) =~ +x 2 Fis continuous on (0, 3). (0) = ~2 and f(3) = 19 -2<4<19 ‘The Intermediate Value Theorem applies. Siete 2=4 Bot tx-6=0 2G x43) =0 xa (2 +x + 3 has no real solution) en2 Thus, /(2) = 4 87. (a) The limit does not exist at. (©) The function is not defined atx = c. 88. A discontinuity at x = cis removable if you can define (or redefine) the function at x = cin such a way thatthe new function is continuous atx = ¢. Answers will vary. "The function is not continuous lim /G) = 1 4 0= tim #6). 86. sa) 222 {s cominvous on f, 4 Te nonremovable discontinuity, Te outside the itera ‘The Intermediate Value Theorem applies. Hires tes 6-6 Bo Sr+6=0 20-3) =0 orx=3 = 2 is nt inthe interval) Thus, (3) = 6. (©) The limit exists atx = c, but it isnot equal tothe value ofthe function at x= e. (@) The limit does not exist atx = c- 1, fee? 0, f-2 2,1isnotan integer 04, 1 C=} 1.08 + 0364 11, 1.04 + 0.36(1~ 2), ¢ > 2, isan integer [Nonremovable discontinuity at each integer greater than or equal to 2. ‘You can also write C as c= fio Ors? 104-0362 - > 2 99, Let s(t be the position function forthe run upto the campsite. s(0) = 0 (¢ = 0 corresponds to 8:00 a.m. (20) to campsite), Let) be the position function for the run back down the mountain: r(0) = k,r{10) ~keo When = 08:00 a4), f(0) = s(0) ~ r(0) When s = 10 (:10.a.4), f(10) = s(10) ~ r(10) > 0. Since f(0) < 0 and f(10) > 0, then there must be a value tin the interval [0, 10] such that /() Section 14 Continuity and One-Sided Limits__78 fla) = a). # 6 then li, they exist) and atleast one ofthese lis equal the corresponding funetion value at £0) = him 0) then des not 94, False; /(1) is not defined and lim x) doesnot exist. 96. The functions agree for integer values of x: a) =3-[-=3-(Cx)= 34% 0) = 3+ bd However, for non-integr values of x the functions differ byt. S13) = 3 + B= 66) ~ a For example, s(!) = 3 +0 = 3, (8) =3 -(-1I) = 4. Jest 98. m9 = 25( 7 9] * 2] 3 [38 Mi) | 50 | 25 | 5 | 50} 25] 5 Discontinuous at every postive even integer. The company replenishes its inventory every two months. Tee ment) kistance 50 ~ rl. Let fl 0.16 /() = 0, then s() ~ 16) = 0, which gives us s(#) = 10). Therefore, at some time f, where O < 1 < 10, te postion functions forthe run up and the run down are equal 16_Chapter 1__Limits and Their Properties 100, LetV = 4277 be the volume ofa sphere of radius r Vis ‘continuous on [1, 5} Va) = f= 49 MIS) = fats") = 523.6 Since 4.19 < 275 < 523.6, the Intermediate Value ‘Theorem implies that there isa least one value r between 1 and 5 such that Vir) = 275. (Infact, r= 4.0341.) 102. Let cbe any real number. Then lim f(x) does not exist since there are both rational and irational numbers arbitrarily close to . Therefore, fis not continuous atc. (© lim san(2) does not exis 106. (@) 702) t pers NOT continuous at x = b. -# xse more ror. =f! {fis continuous for x < c and for x > ¢.Atx = ¢, you need | — & = c, Solving c + ¢ ~ 1, you obtain ewrieviFd_-12 5 2 2 101. Suppose there exists in[a, 6] such that f(x) > 0 and there exists x in [a b such that f(x.) < 0. Then by the Intermediate Value Theorem, (2) must equal zeo for some value of xin [xx] ot [xx] fxg < x). Thus, f would have a zero in (a, 6), which is a contradiction, ‘Therefore, f(x) > O for allx ina, 6] or f(2) < 0 for all xin [a, 8) 103. If = 0, then f(0) = O andi fla) = 0. Heme, fis continuous ats = 0. It #0, then tim (0) = 0 for xrational, whereas Jim 0) Jim kt = kx #0 for x rational, Hence, fis no continuous for allx # 0 108. (@) peeve (&) There appears tobe a limiting speed and a possible cause i ar resistance, osxsb ©) gt) bexsm ‘Continuous on [0, 25]. 108. Let y bea eal number. Ify = 0, then x = 0. fy > 0, then Tet 0 < xp < 17/2 such that M = tan.xp > 9 (his is possible since the tangent function increases without bound on [0 7/2). By the Intermediate Value Theorem, 0) = tan.xis continuous on (0, x9] and 0 < y < M, ‘which implies that there exists x between and x such that tan x= y. The argument is similar ify < 0 Section 14 Continuity and One-Sided Limits 7 10. fo) = F=E=S co 0 Domain: x + 2 0 => x2 —c and x # 0, [-c*,0)U(0, 00) in EEE HE oy EEOC, ETE He Bg i ede @ra-e2 = lim May ard+d Me yetdre Define (0) = 1/(2c) to make f continuous at x = 0. 110. 4. fle) is defined, 111, HG) = a 2 lim (2) = Jim, fle + 3) = f(0 exis. (Letx = 0+ Ax Asx6, 4x0] 3. ling) = 400. fs ‘Therefore, fis continuous atx = ¢. t ‘has nonremovable discontinuities at x= 4142.23, 112, (@) Define f(s) = 4(2) ~ fl). Since f, and f, ae continuous on [a 8}, 0 sf la) = fa) ~ f(a) > Oand fb) = fs) ~ f(b) < 0 By the Intermediate Value Theorem, there exists in [a,b] such that fc) = 0. 10) = f(0) ~ fe) = 0 => KO = HO () Let f(a) = xand ffx) = c08 x, continuous on (0, 7/2} f(0) < f(0) and f(m/2) > f3(n/2). Hence by pat (a), there exists in [0, 2/2] such that © = cose) Using a graphing utility, ¢ ~ 0.739, 113, The statement is te. Ify = Oandy < 1, then y(y ~ 1) < 0 < 32, as desived, So assume y > 1. There are now two cases. Case 1: Ix s y~ $,then 2x + 1-5 2y and case: fx 2 y-4 Hy = 99 +1) 29 ve(y-f 0.) © im, [S0meec? 6] = 00 © Hy (S0msec* 6} + Sg ee 60. c= EAE < x < 100 (@) €(25) = $176 milion (© C(5) = $1584 million (©) (50) = $528 million co. Thus, it is not possible. 61, m= — = 07S) Jim m = tim —— 2 ane ln 7 oe ‘Total distance Total time fx [20 | [50] 0 2d 30 | 42.857 a+ yee 50y + 50x = 2x9 SOr = 2ay ~ Soy 50x = 2yfx ~ 25) 25x ‘As x gets close to 25 mph, y becomes larger and larger. ex > 25 Section 1.5 _Infinite Limits 83 * OT Tt [os | 02 | 1 | 001] oor | ooo 76) | oases [ 008 | 00067 | oo017 | =o | =o | =o © [1 [es [02 [01 | 001 | 0001] ooo Fee) | oases | 0002 [ 00333 | oors7 | ooor7| =o | =o © [1 [es [02 [01 | 00 | 0001 | 00001 Fea) | oases | 01646 | 0.1663 | 0.1666 | 01667 [0.1657] 01667 oul 7 = 0.1667 (1/6) fF [a [0s | 02 | 01 | om] oon] ooo ie) | 01585 | 05292 [ 08317 | 1.6658 | 16.67 | 1667 | 16670 65. (a) A= 20m — 420= {10,10 tan) - 4c10)20 © ah em a x @ [03 | 0s] os [ia] is = s0tan 9-500 1@)| 0.87 | 421 | 180 | 686 | 6301 oe @) lim, A= 00 oth 84 Chapter 1__Limits and Their Properties (6. (a) Because the circumference of the motor is (©) The direction of rotation is reversed. half that ofthe saw arbor, the saw makes a 1700/2 = 850 revolutions per minute. | 03 | 06 | 09 | 12 | 1s (© 220 cot 4) + 2(10 cot f) sraght sections. 1 | 3062 [ 2179 | 1959 | 1996 | 1885 ‘The angle subtended in each circle is oe (lf-4) ae "7 ‘Thus, the length of the belt around the pulleys is 20(m + 24) + 10(a + 24) = 30m + 24) o a Total length = 60 cot $ + 30 + 24) © tu, 1 = Gon 1885 (All the belts are around pulleys.) @ fig.t= 20 ‘The graph of fhas a hole at (1,2) not a vertical asymptote. 70. Fae fl, soy {3 ‘The graph off asa vertical asymptote atx = 0, but £00) = 3. x#0 x=. 72. Given tim f(x) = oo and lim g(x) = Le @) Product: TEL > 0, then for ¢ = L/2 > Other exist 6, > O such that (a) ~ Z| < L/2 whenever 0 < |x ~ cl < 8. Thus, L/2 < (x) < 3L/2 Since lim f(s) = 20 then for M > 0, there exists 8, > O such that f(x) > M(2/L) whenever lel < 8, Let Bbe the siller of 8, and 6,. Then for 0 < [x ~ o| < 8, we have f)e(x) > M(2/L)(L/2) = M. ‘Therefore lim /()g(2) = 20. Te proof is similar forL <0. ) Quotient: Let « > Oe given There exists 5, > Osuch that f(a) > 3L/2e whenever O < [x ~ c| < 6, and there exists 6, > O such hat [g(X)~ Z| < L/2 whenever 0 < fx cl < 8 This inequality gives us L/2 < g(x) < 3L/2. Let 6 be the smaller of 8, and 6, Then for O 3. Let M > be siven, We need 8 > O such that f(s) whenever3M ‘hiss ot posible Thus, im fs) does not xi Equalenty.x 3 < 2 wheoer 1 So take 8 = Then for x > 3and |x ~ 3] < 8 Le > stg > Fe Mand hence (0) > at 76, fa) = <1 jis defined for all x < 4, Let < Oe given. We need 8 > 0 soch that fe) = Equivalently, x — 4 > 7+ whenever |x ~ 4 < 8, < 4 Bquivalently, whenever [x ~ 4] < 8.x < 4, Sotake O because < 0, For fx~ 4] < Bands < 4, tp Review Exercises for Chapter 1 1. Calculus required. Using a graphing utility, you can estimate 2, Precalculus. L= J@= I+ @— 1} ~ 825 the length to be 8.3. Or, the length is slightly longer than the distance between the two points, approximately 8.25. 4 3. fo) = 22 | = [-01 [-oo [-oon | oo | oo | ot 70) | — 1.0526 | -1.0050 | ~ 1.0005 | —09995 | 09950 | -09524 sim 7) “Tx [-o1 [-oor [=o00 | oo [001 [01 + z c ae ye) | 1432 tate | aia | rai [1403 | 1397 7 7 tim fl) ~ 1.414 Bo2e 5. 1g) = 2 (Lig nb) (@) im (2) does not exist. ©) im, #0) = © lim g(a) = 0 7. lin (3 ~ 2) =3 Let © > Obe given. Choose 8 = «.Then for < |x ~ I] < 8 = e,you have be-tlee [nace I6-»-21 <6 [f@) -L]) <« 86 Chapter 1__Limiis and Their Properties 8 tin Jem = Late > Obe given, We need [ve-3] [Ve4 3] ve—3] Obe given. We ned #3 — 1] < 6 => i — 4) = [b6— 20 +2) <= fe 2)-< ‘Assuming, 1 < x <3, you can choose 8 = 4/S.Hence, for 0 < |x ~ 2 < 6 = «/Syou have . bom efe eta bre +2] <6 bed] <6 Ie@-3)-I] Obe given. can be any positive 11 lig VFB = VETS = Jo ~ 245 number. Hence, for 0 < |x ~ 5] < 6, you have -9 —492 + 200=0 = = M0816 = 1 = 639 500 ‘When a = 6:39, the velocity is approximately ae = Jim, -49@ +) —4.9(6.39 + 6.39) 62.6 m/sec. 33. lim s(a) = 0 dim W = 141 = 2and fim Mo) =H +0) = 37s) = b+ 3} cig, b+ 3] = & + 3 where kis an integer Tim be + 3] = & + 2 where kis an imeger. [Nonremovable discontinuity at each integer k Continuous on (kk + 1) for all integers & 32 Gxt 2-1) x1 xo 1 im fG) = lim Gr +2) = 5 28. #6) = Removable discontinuity atx = 1 Continuous on (—20, 1) U(1, 20) 40) = Gp 1 AGF Nonremovable discontinuity at x = 2 Continuous on (20,2) U (2, e8) 810) lim f00) = =o0 “in (0) = 2 Nonremovable discontinuity at x = ~1 ‘Continuous on (—20, ~1) U(“1, 20) Gx+DUe- i) eT Fis continuous on (—20, 1) U (1,60). wos fO8, 255 Jim, (2x = 3)= 1 "Nonremovable discontinuity at x Continuous on (~20,2) U (2,20) aa. soy> fFtte ist Ha Domain: (—c0, — 1}, (0, 00) Nonremovable discontinuity at Continuous on (~00, ~1]U (0,0) +1 +2 stl oa MIG) 2 4. £0) = Removable discontinuity atx = =1 Continuous on (~20, =1)U (1,20) Review Exercises for Chapter 1 _ 89 45, f(a) = o3e F 46, fx) = tan 2x 47. $02) = ‘Nonremovable discontinuities st ‘Nonremovable discontinuities when Find eso that tim (cx + 6) = 5. each even integer. (n+ I) 0) +6=5 Continuous on + ; (24,28 +2) continuo on aa (2rpbs Gan) 7 for ll integers 48 lim (c+ 1) =2 lim@+)=4 Find b and cso that Jim (2 + bx + c) = and lim, G? + be + 6) = 4 Consequently we get 1+b+c=2 and 943 4em4, Solving simultaneously, and en4 49. fis continuous on [1,2).f(1) = ~1 < Oand 50. C= 980 + 2.50{-[-x] - 1], > 0 ‘f(2) = 13 > 0. Therefore by the Intermediate Value “Theorem, there is atleast one value cin (1, 2) such so- 250-4 +1] P thar2e 3 = 0. Chas a nonremovable Aiscontnuty at each imteger. 82. ft) = Ve Te (@) Domain: (~c0, JUL, 2) ©) Jip 0) () im, 76) = 4 al ms © Jim so) (© ims) doesnot exis. 83. 9) =142 54. hs) = 85.10) = Foe ‘Vertical asymptote at x= 0 Vertiealasymptotes atx = 2 and Vertical asymptote at x = 10 ye? Betxt 36. fla) = ese mx Se Vertical asymptote at every integer k 90 Chapter 1__Limits and Their Properties (@) CU1S) = $14,117.65 (6) C(50) = $80.000 - 80,0009, (© 090) = $720,000 (li, $9008. 70, fa) = S22 OT 01 0.01 0.001 | 0.001 | G01 OL SG) | 2.0271 | 2.0003 | 2.0000 | 2.0000 | 2.0003 | 2.0271 a (©) Yes, define _ faze x#0 wf "Now fla is continuous a = 0. Problem Solving for Chapter 1 1. (@ Perimeter APAO = JE FUT TF + VER +1 2.) Area APAO = JETER + EFFI Perimeter APBO = V@— IF Fy + VET +1 ‘Ars APBD = EAT + HL or) 22 + + a) * ‘area APAO ~ x/2 ©) ra) = ZR + VE Vani e+ Jee 41 = 4[2[ 1 [a1 [oo x 4 [2] 1 | 0 | oo ‘area pao | 2[ 1 | 1/2 | 1/20 | 1/200 Perimeter Area APBO | 8 1/2 | 1/200 | 1/20,000 Apso | 3302 | 9.08 | 3.41 | 2.10 | 201 atx) 4 1} 1/10 1/100 Perimeter P60 | 33:7 | 9.40 | 3.41 | 2.00 | 2.00 (© io = Si, ria) 0.98 | 0.95 | 1 | 1.05 | 1.005 3. (@) There ae 6 triangles, each with a central angle of 60" = m/3. Hence, | -Aho = 28 ~ 2598, ‘Area hexagon LN en Bre 2-34 ~ osas neal meee azn] =n] mitt. fn [6 [2[ m | % | 96 306 [ 3.133 | 3.139 A, | 2598 | 3 (© Asn gets larger and larger, 22/n approaches 0. Leting x = 2n/n, = sin(Qa/n) A Un Which approaches (I) =, sin@a/n),_ sinx, =" Gain) 7x 5. see 2 ©) Slope of tangent ine is tin 2 VIB=F 12+ VIF x= 5 Re Vie—= = tim 14 = 1 — 2) IRG= 92+ ea a2 BG siz + 18 = x) = tim —2*9. at Jee ‘This isthe same slope as part (t). Problem Solving for Chapter |_94 4. (@) Slope = 9 =F o) supe = 2 tngatine y= 4= 2-3) tim —G—9G+2) 3 (x — NVI — x + 4) B+) “aa 4 ‘This is the slope of the tangent line at P. 6 VERB _ aT aT + x x Vath 3 a+b) Vat ie Letting « = 3 simplifies the numerator. Thus, i EEA Vg ad = wi /3 + bx + V3) = tim —— =I ares VF » ting P= V3, you obtain b= 6. Setinn Tp Zs ~ V3 300 ‘Thas,a = 3andb = 6. 92 Chapter 1__Limits and Their Properties 1 34+x220 o xB 2-3 xe Domain: x 2 -27,x #1 AER 2 EF 42 * VERB +2 tg Bt aa) eha1 © tims) = tim GR De Pe VST) = lim 4 in aa pees Fe) 1 “GFT naFD 1 -$ im 76) = im (@? ~ 2) = a? = 2 ip. 76) = Jin <2 = a (mecanse tm 82 ~ 1) Thus, 2=a @ 2=0 (a-2a+=0 a=-12 @) #Q) = I= 4 £0) =f] =0 f0) =f} =1 ©) Jim fe) = 1 lim f0) = 0 lim f6) = ~20 ‘lim £6) = 00 (© f is continuous for ll eal numbers except 9. (im) = 3: exes ( Feaninwous 2:5, © jim fa) = 3: gis Be © ims) lin, £6) lim, £0) = -1 (© Fis continuous for al real numbers except x= 0,41, 42,23, Problem Solving for Chapter 1 © @ Jim Pay) =1 Gi) Jim P46) = 0 1920 stone QUT 1 (©) P58 continuous for all positive real numbers Foy + aT acer aTats nan) (@) The area under the graph of U, and above the 2 axis, is 1 Let vy = JET mise (= 1.47 mised), ype tous. OR OB 10,600 99 = ve Let) = 655 = 2.64 mi/see. ‘Since this is smaller than the escape velocity for Earth, the mass is less. 14, Leta # O and let © > Oe given. There exists 8 > Osuch that if 0 < |x ~ O| < 6 then [f(x) ~ | < Let 6 = 6,/lal. Then for 0 < |x ~ 0| < 8 = 6,/lal, you have 4 “Tel axl < 8, [flax) ~ 1] < & As a counterexample et a = O and fs) = tf: ay ‘Then lim f(s) = 1 = Z, but lim fax) = lim f(0) = lim 2 93 CHAPTER 2 Differentiation Section 2.1 The Derivative and the Tangent Line Problem... . . 95 Section 2.2. Basic Differentiation Rules and Rates of Change. . 109 Section 2.3 Product and Quotient Rules and Higher-Order Derivatives ..........00204 120 Section 2.4 The ChainRule ...........0... oo Section 2.§ Implicit Differentiation ........... 147 Section 2.6 RelatedRates ............ cee ee OL Review Exercises 6.2.0.2... 022.00. cee ee I Problem Solving... 2.6.0... 0-0 eee eevee eee 184 CHAPTER 2 Differentiation Section 2.1 The Derivative and the Tangent Line Problem 1. (9 At Opa) slope = 0. At raph slope =. ©) Atl yp slope = At G9 slope = 2. 3. @.0) Bey enp=ett © y= =e es 3 =Fe-n+2 =I@- +2 ext 1. Sopeat (3) = fi #82 = a) im (Lt As = = (23) mo ay m2 + Qo] =2 9, soe (0,0)= 0 80 =L0) tn 1 28x) + (Oa)? = 1 2, (a) At (3,9), slope ~ 3 At (9) slope = (©) At slope = At (439), slope ~ £0) =f) ,8= 478 35 £4) =f), $@) - £0) Ts GTP aad (©) The slope of the tangent neat (1,2) equals (0). ‘This slope is steeper than the slope ofthe line through (1,2) and (4,5). Thus, 28229 FQ) = 1 Matches ©) 38. fle) = > fe) = 2 Matches. S00) = JE 2 $0). Matches 40, f° does not exist ax = 0. Matches (6). (Geereasing slope as x23) 4, (6) = 2 because the tangent ine asses tough (5,2). 42, {~1) = because the tangent ine passes through (—1, 4 9955-3 wena Sy dn3 48. The slope ofthe graph off 44, The slope ofthe graph of | 48, The slope of he graph offs bl oye=1 is0 > 7) -0. neptive for x < 4, postive forx > 4,and atx = 4. 102 Chapter 2__ Differentiation 46, The slope of the graph of, forx < 4,1 forx > 4,and undefined at x= 4. 49. flx) = 5-3 ande= 1 56. (a) If F(e) = Band f is odd, then fe) = fe) = 3. 57. Let (x3 be a point of tangency on the graph off. By the limit definition for the derivative, f(x) = 4 — 2x. ‘The slpe ofthe line through (2,5) and (9) uals the derivative of fat Sy Sonat S~ y= @~ sgl 2x) 5 (4x9 — 397) 0 = (59 ~ tty ~ 3) 2 x9" 1,3 ‘Therefore, the points of tangency are (1, 3) and (3,3), and. the corresponding slopes are 2 and —2. The equations of the tangent lines ae: yr 5= 2x2) yard 47. Answers will vary, ‘Sample answer: y 51. fla) = -atand e= 54. f00) = 4,70) 1 £10) < 0 for 2 <0,f'(2) > Oforx > 0 x#0 fo)= +4 48, Answers will vay. Sample answer: y =x 52. fx) = 2Ve and c= 9 58. f(0) = 0;F(0) = 0:/13) > Oit fa) = 2 (©) IFC) = 3 and f is even, then p(=c) = =s'(e) = =3 ‘58, Let (xo, yo) bea point of tangency on the graph off. By the limit definition forthe derivative, (2) = 2x. The slope of the line through (1, ~3) and (x 4) equals the derivative of fat x: 2 2x T% ~3— Y= (I~ 22) 3 ~ 4g = 2x — 2? 2-2 - x 9 ~ N9 + 1 ‘Therefore, the pont of tangeney are (3,9) and (—1, 1), andthe comesponding slopes are 6 and 2. The equations ofthe tangent lines are: Se3-1 Section 2.1 The Derivative and the Tangent Line Problem _ 103 $9. (@) (0) = -3 ©) &)=0 (©) Because ¢'(1) = —5, g is decreasing (falling) at (@) Because ¢(~4) =, gis increasing (rising) at x = ~4 (€) Because ¢() and g'(6) ae both postive, (6) is greater than g(4), and (6) ~ g(4) > 0. (f) No, itis not possible. All you can say is that gis decreasing (falling atx = 2. 60, @) $6) = © et) = fim, A= @eay-2 a a tn EE BAA) + Aas + (Aa = as, ‘ar im A222 + Bald) + (499) tim Sx@x+ Bx) a ar “a ar = Jim, Gx? + Sxl) + (A) = 3 = fim, Qe + As) = 2 “ Atx= —1,g(-1) = 3 andthe tangent ine is Atx= ~L (CD) = ~2 and the tangent line is yele seri) or yaaet2 =H2r+ 1) or y= ~~ 0,40) = O.nd the tangent ine is y = 0. and the tangent line is. and the tangent line is. .@°(1) = 3 and the tangent line is yr 13-1) or yaar, For ths function, the slopes ofthe tangent lines are always distinct for different values of x. For this function, the slopes of the tangent lines are sometimes the same. 61. f@) = be 2 By the limit definition ofthe derivative we have (3) = 32 [ » [=] -5[-1] 05] eos] 11s [2 I 10) | -2| -8 4 pols (:l#P + rol | % & [ol# [2] #[s 62, fla) = fa? . By the limit definition ofthe derivative we have f(s) = » [-2[-1 [-1 [-05 [ojos [1 [1s [2 4| ? 7@ | 2) 1s] os] ois [o| ois | os | 1s [2 > SG) | -2 | -15 -1 -05 0} os 1 1s 2 108 Chapter 2__ Differentiation fle + 0.01) ~ fle) fle + 001) ~ sls) 63. glx) 001 64. a(x) O01 D(x + 0.01) ~ (x + 0.01) ~ 2x + x7]100 101 a2 ‘The graph of g(x) is approximately the graph of f(3) = 2 ~ 2x 65. f(2) = 24 = 2) =4, 21) = 2114 = 21) = 3.99 ‘As x00, is nearly horizontal and thus f”= 0. 0. $0) = 4- = 3 Sue) = LEAD ALE) G5 29 + 0) =@+dr-38- 3, ge = te 2) +3 1 Sy, 1- (ar @ are (e-2)4+3—x41 r= 05: Sue (S)r-2)+3 =» ar (sVe¥00T - 3V3)100 ‘The raph of approximately the gap of) = 52 64 12) ~ So) = 2,21) = 23s 231525 = 2 21-2 FO) = 31525 [Bxact: (2) = 3] -2)+3= Ax + 2Ye— 2) +3 arson 5y=(Qe-n+3= Bef Va (6) As Ax-70, the line approaches the tangent line to fat (2,3). 70. 70) =x 4 Los @+ag+zt-5 = 12 +89- JO. - 23h 2,45 Sac (a) = LEAD L0) 6, — 9) + 912) = i e+ RH 2RH AP +2~SO4+AD, 4 ,S_ Carty 5 waar anas 89+ 2~2@ Fay 749 3 2 oe Beg 16, Ha-a+d=3 (©) As 4x70, the line approaches the tangent line to fat (2, §) Section 2.1. The Derivative and the Tangent Line Problem 105 nf) = = f(x) = f(2) 10) = ti Oe c=? 72. g(x) = x(x - Ie —He=1 st0)= tm =O = tn BiG) = 42+ Lew -2 14, fl) = 8 + 2x0=1 10) = ty APO = yy 2 E23 = py 8 ESAS D «nero x 3) =8 15. ea) = Ve = 0 ane Ba ah oc, rox 07 Ela ton 76.50) = 4, PG) = tim L2=L2) _ Be 3 jy 1/9) x23 ARG Th. $0) = («- 6F?,0 = 6 106 ~ ig L2=L9-— tn Does not exist. 78. glx) = (x + 3)"e= -3 7 =e). #9 = fin, =CH oes not exist. @+3-0 x43 Ge ape 9. hls) = |v + Sl,e= -S 5 = tn MIEMED)— pg ESL es) HS) = tin oD MM eS aS oes no exist. 80. fb don 4 f(x) is differentiable everywhere except at x = —1. (Discontinuity) = im P= laa oes not exist. 106__ Chapter 2__ Differentiation £82. fx) is diferente everywhere except at x 83. f(x) is differentiable everywhere except at. (Sharp turns in the graph) (Sharp turn inthe graph) 84, f(x) is diferentiable everywhere except atx 85. fx) is differentiable on the interval (1,20). ‘iscontinuties) (Atx= 1 the tangent line is vertical.) 86. f(x) is difereatable everywhere except at x (Discontinuity) 87, f(x) = [x + 3] isitferetiable 88. fe) = a el forall # 1. f isnot defined a x= 1. (Vertical asymptote) 90. f is differentiable for all x # 1. 91. fle) = {snot continous atx = 1. “The derivative from the leftis 1 tim LY=L0 jig R= = 0 ABT gay MB SRT k- a] I ‘The derivative from the right is fig LO=LD yy, B= =O, ABs ARO SRT ‘The one-sided limits are not equal. Therefore, differentiable atx = 1. 92. j) = JI=# ‘Te derivative fom the let doesnot exist because £0) = f0) . Lee et fin LO in ~ Big fetes = ~e0. (Vera! angen) ‘The limit from the right does not exist since fs undefined for x > 1. Therefore, fis not differentiable at x eax poy {Ores ‘ie devvative om te ets tig LL jpg =P ART ST Spe ‘The derivative from the right is tin 2-20) tT ‘These one-sided limits are equal. Therefore, fis differentiable atx = 1. ((1) = 0) Section 2.1__ The Derivative and the Tangent Line Problem “The derivative from the left is tim L=L0) hg © rr = pict “The derivative from the right is tip QED = tn FSP a sD ‘These one-sided limits are not equal. Therefore, fis not differentiable atx = 1. et hxs2 Jar 3, x >2 eda fon tees tn LO=L2)~ PADS in oa = “The derivative fom the ight isi, £22 £2) — fy SEBS im = 4, 95. Note that fis continuous at x = 2. f(x ‘The one-sided limits are equal. Therefore, fis diferentiable at x = 2. (12) = 4) “i detix<2 96, Nate tt jiscoimour atx = 2.0) = [EEE XS? Teds ome ti Bg BEAD fy BNR yg ED ‘he drat fom he ii tim LO=LO) yy, YER? VHD Bp ya2 7 MBA r 2 Vert? 2x4 2-2) 2 ‘The one-sided limits are equal. Therefore, fis diferentiabe atx = 2. (12) = §) 97. (a) The distance from (3, 1) to the line mx ~ y + 4 = Dis las, + By, +c ga Met Bn + ol Tae (3) ~ 101) +4) [3m +3) Tie” ST © * “The function dis not differentiable at m = ~ 1 This corresponds to the line y= —x + 4, which passes through the point (3,1). 107 108 Chapter 2__ Differentiation 98. () fle) = and fta) = 2x (©) els) = xP and g's) = 3x2 (© The derivative isa polynomial of degree I less than the original function. If h(x) = x, then Wa) = mer £0) = ton Bt AAs) + 6a)? + dha)? + (Aa) — xt a re tim AMG + 61s) + du(ds) + (A) 2B ar fim, (4x? + 6x2(A2) + Aa(a)* + (Aap) =a Hence, if fx) = x4,then f(x) = 4x? which is consistent withthe conjecture. However, this is not a proof since you must verify the conjecture forall integer values ofn, x2 2. 99, Fs Te seis fin, £2 A=) ton, Tae.» ~ f= 2 conn a = 2, oe nt ae e differentiable at x = 2, (Sharp tum in the graph) 101. False. I the derivative from the lft of a point does not equal the derivative from the right of ¢ point then the derivative doesnot exist at that point. For example, i/(x) = [| then the derivative from the Jeft at x= 0 i ~ 1 andthe derivative fom the right at x = 0 is I. At.x~ 0, the derivative does not exist. 102, True—see Theorem 2.1. 103.6) patios x40 x=0 ‘Using the Squeeze Theorem, we have ~[y| < xsin(I/s) s [pl, x # 0. Thus, im x sin(1/3) Fis continuous atx = 0. Using the alternative form ofthe derivative, we have * ig BO - ty EL? talons) Since this limit doesnot exist (sn(1/2) oscillates between —1 and 1) the function isnot differentiable atx = 0. fx*sin(i/a), x #0 x20 2G) Using the Squeeze Theorem again, we have ~32 < x2 sin(I/x) $ 22, # 0. Thus, lim 2? sin(1/x) ‘and g is continuous st x ~ 0. Using the alternative form of the derivative again, we have tig f= HO) py 8/0 nit ‘Therefore, gis dferentable atx = 0, (0) = 0 Section 2.2 Basic Differentiation Rules and Rates of Change _109 104, 2 'As you zoom in, the graph of y, = 3° + 1 appears tobe locally the graph of «horizontal line, whereas the graph of Jy fx] + Halas has a sharp comer at (0,1). yx isnt difeentiable a (0, 1). Section 2.2 Basic Differentiation Rules and Rates of Change L@ ya" © yee 2@ ® yart yee yere? yO=4 y@an1 BR ya8 Ss yaah 6 yaa yim 6x! a 10. y= f= x0" pe dyn et ve " act 11. 6) re) Ma ye + 2-3 yews? 18. gs) =x? + 4x? eG) o 2x + Ie 18, f0) = 28 = 8 +a 50 PG) = 62-2843 sin@— cos 20, g() = meas 2 y=S+sinx ey 9) = ~msin cose Function Rewrite 5 3 2 ye Se yade? 2 26 y= 55 3 Wy oR 110__Chapter 2__ Differentiation Eunction. ma ym oe Diya L0)= 6 30° ~ 6, (2,18) yon ot y'@) = 36 31. f(@) = Asin O ~ 8, (0,0) xO) ro cos = 1 1) - 40, f(9) = x2 ~ 3x3? SI) = 28-3 + 6x =2r-3+$ 4a fy PoE 4 yaa ev tx 24d Sa) = Vi ~ 625 So) =p ae 49, He) = 45 — 92 = 4yeus 25-1 Wi) = 308 — Fe 38. y= Ox +1 0,1) ae bart Bets yO=4 38, 9) =2 + 3coss, (m1) (= —3sine e(a=0 41-04-04 eO 924 art ma 36, f(x) = 305 — 2), (5,0) ee SG) = 6-30 s)=0 P45 — 3x? 39. f(x) payne orate $ 2 fos)axt x? PG) = 1-209 46, y = 3x(6x ~ 5x2) = 1837 - 153° y= 36x = 4502 48, J) = YE + Y= WH + x88 1 fa) =F + ee -toud ae * eas 50. f() = PO =D 4 3 r= 2 Bye = ad 3a Section 2.2 BL. fla) = 6/ + S.cosx = 6x"! + Scosx 3 Ha — Ssiny = 42 — Ssinx re) = 3.) yout 3+? yrs at = 6 At(1, 0): y’ = 4(1? ~ (1) = ~2 ‘Tangent line: y= 0 = -2x-1) 2 +y-2=0 o F | ua 55. (a) fx) At £0) ‘Tangent line: o 020 Oforallx ‘Therefore, there are no horizontal tangents. Atx= Horizontal tangent: (0,1) 12 Chapter 2 _ Differentiation OL y= xt sing, 0 Sx < 29 yim Lt eosx = 0 cosx= lap xem Aue Horizontal tangent: (x, 7) 68. x7 — ke= 4x9 quate functions. 2e-k=4 quate derivatives. Hence, k= 2x ~ 4 and x= Qe 4) Forx = 3,k=2andforx = -3,k= 4x— Sap x? 9p 3, 10. Equate functions. 6, (a) The slope appears to be steepest between A and B. (©) The average rate of change between A and B is greater than the instantaneous rate of change at B. oO; 69. glx) = fla) + 6=9 8) = Ft0) @. y= Jie + 2eosx, 0S x < 28 V3-2sinx=0 Bo Bon 2 Fegng = y_V3nt3 po de aJin-3 onan: ($282) (28 2583) ee ee eee O forall xand he rte of change the funtion is decres- ing (ie, #” < 0) would in general lok like the raph at —_ the right. 70. g(x) = ~5f0) => 2) = ~570) Section 2.2 __ Basic Differentiation Rules and Rates of Change 113 If fis linear then its derivative is a constant function. If, is quadratic, then its derivative i a linear function. 0) =ax+b $0) =a tbe te se) =a fe) = ax +b 73. Let (xy,3,) and (4,9,) be the points of tangency on y = x?and y = x4 + 6x ~ S,respectively. ‘The derivatives of these functions are: ye 2x m=2x, and y= -2x+6 > m m=2x, = -2x +6 at atom 3 Since y, = xand y, = —x48 + 6x, ~ 5; (ox? + 6x, 5) ~ (g? = 6x; +9) = (2m, + 2, ~ 3) 2x? + Idx, ~ 14 = 4x2 + 18x, ~ 18 2x? - 6x, + 4=0 2s — Ale — I) = 0 x= lor? Dandy, =4 ‘Thus, the tangent line through (1,0) and (2,4) is (Aen vara ant a = Landy, = 1 “Thus, the tangent line through (2,3) and (1 1) is Gae-n= 2e- 1, 74, m; isthe slope of the line tangent toy zm, i the slope ofthe line tangent to y = 1/2, Since yoraey > “1 me ab my The pois tinteneion oy = xan = sae robe tet as seat Atx= hm, 1, Since m, = —1/m, these tangent lines are perpendicular atthe points of intersection. 114 Chapter 2__ Differentiation 78. fs) = 3x + sinx +2 16. fle) = x8 + 3x + Se £0) = 3+ cosx $10) = 5x4 +982 +5 Since cos x] < 1, £10) # 0 for all x and f does not have Since Sx* + 9x? = 0, /(a) 2 5. Thus, f does not have a ‘horizontal tangent line. ‘tangeat line with a slope of 3. 7. fh) = Vi, (-4,0) Se) = be" ee 2a =4— 44 x= 2Viy Aten Wave 4txee xm Aya2 ‘The point (4,2) ison the graph off ‘Tangent line: y — (r= 4) “The point (f) ison the graph off The slope of te tangent ine) = —$. ata ‘Tangent line: m. FA) 0. (4) =1 ‘81. (a) One possible secant is between (3.9, 7.7019) and (4, 8) 8-709, yo 4) fa y= 8 =2981(x - 4) “ce 7 Sx) = 2.9814 ~ 3.924 10) = 327 = 7 =3a)=3 2 TG) = 3-4) +8 = 3x— 4 ‘2) isan approximation ofthe tangent line T(x). —CONTINUED— Section 2.2 ‘81, —CONTINUED— Basic Differentiation Rules and Rates of Change us (© As you move further avay fom (4,8), the accuracy ofthe approximation T gets worse. , Ola -3] -2 -1 -05 | -O.1 0} o1 0s 1 2 3 ‘4+ hx) | 1] 2828] s.196 | 6548] 7.702 | 8 | 8.302 | 9.546 | 11.180 | 14.697 | 18.520 m4+ dx) | -1] 2 5 65 77 -|8}83 jos ju 14 17 ‘82. (a) Nearby point: (1.0073138, 1.0221024) (&) £@) = 37 - (ae +1= 32 secant ine: y ~1 = MORO =e) mane) . (©) The accuracy worsens as you move away from (1, 1). y= 3.022 - 1) +1 2 (Answer wlan) i 7 © [ar [-3[-2]-1] -0s -O.1 0} 01 Os 1] 2] 3 fa) | -s]-1] of 012s] ono [1 | 1331 | 3375 [8 | 27 | 64 ‘The accuracy decreases more rapidly than in Exercise 81 because y = x° is less “linear” than y = 2%, 83, False. Let f(a) 2 and g(x) = 32 + 4.Then JG) = #°) x, but fa) # eta). 85. False, fy = 2, then dy/dx = 0. (ais a constant.) ‘True. If g(a) = 3/0), then ge) = 3/0). 89. (= 20+ 7,1,2] s=2 Instantaneous rate of change isthe constant 2, Average rate of change: £2)=$0)_ 22) +7]- 20) +71, 2-1 T (These are the same because fis a line of slope 2.) 84, True, If fC) = a(x) + 6, then f(a) = @'(e) + 0 = 2G). 186, True, Ify = x/m= (1/n) “x, then dy/de = (X/)(1) = 1/. ‘88, False. If f(y) then f"a) = —nx-tt = 9. fi) =F ~3, (2.2.1) re Instantaneous rate of change: @,1) => f@)=2@) =4 Q4,1.4 > P41) =42 Average rate of change: £21) = 40) 21-2 116 _Chapter 2__ Differentiation Instantaneous rate of change: (,-) = f= 1 1 (0-3) 410-3 Average ate of change #2) = s)_ 1) 1 2-1 2 2 93. (a) sf) = — 1607 + 1362 vi) = ~320 © a2. 1998 ~ 1346 = ~ 48 8/5ee 31 1: v(1) = —32 ft/sec ‘When t = 2: v(2) = —64 ft/sec ( ~162 + 1362 =0 2 BO _, , VBB_ a BSL, YE ose <8) 8 VISE ~ ~295.242 f/see 95. si) = -499 + vot +5 = 4902 + 1207 (5) = -9.8(5) + 120 (10) = ~9:8(10) + 120, 97. From (0, 0) to (4,2), s() = $¢ => v(0) = }mi/min. v4) = H60) = 30 mph ford. <1< 4 ‘Similarly, v@) = 0 for 4 < ¢-< 6, Finally, from (6,2) to (10.6), 4 = (i) = I mifin = 60 mph. ] 9. fla) = sinx, 4] J'3) = cosx Instantaneous rate of change: (0,0) => s'(0) =1 (33) =) om 6 6 Average rate of change: Lx/6) = f10) _ /2)~ 0 _ 3... goss (x/6) — (a/6)- 07 4, s() = — 161? — 221 + 220 wi) = 32-22 v3) = 118 ft/sec sf) =~ 162 — 221 + 220 12 (height ater ating 108 ) — 164 — 22+ + 108 —2(t — 28" + 27) 2 (2) = —32(2) — 22 = ~86 ft/sec ASP + vob + 5 4912 + 55 =0 when 1= 68. 91 = 49(6.8) = 226.6 m 96. s(0) ety Gy Levers ‘Tieemie) (The velocity has been converted to miles per hour.) Section 2.2 99. y= 40 mph =} mi/min (3 mi/min)(6 min) = 4mi y= Omph = Omi/min (Omi/min)(2 min) = Omi v= 60.mph. (Q.mi/min)(2 min) = 2 mt ie Gn si/min 101, (@) Using a graphing utility, = 0417 = 0.02. () Using a graphing utility, B= 0.005570? + 0.00140 + 0.04. (© T=R + B= 0.005570? + 0.418 + 002 @ * 102. C= (gallons of fuel used)(cost per gallon) Basic Differentiation Rules and Rates of Change 117 4100, This graph corresponds with Exercise 97. Tie eae) co B= oon +a For v = 40, T(40) = 0.86. For v = 80, 7(80) ~ 1.31, For v = 100, T(100) ~ 1.53. (f) For increasing speeds, the total stopping distance x | 0 | 1s | 2 | | 30] 35 | 4 € | 2325 | 1550 | 1163 930 | 775 | 664 | 581 ac | -233 | -103 | -s8] -37| -26] -19] -15 ‘The driver who gets 15 miles per gallon would benefit more. The rate of change at x = 15 is larger in absolute value than that at x = 35. 48 cm per em change in s. =n 14. A= Whens = 4m, 2s 4s qae mte pert anein 118 Chapter 2__ Differentiation 105, s() = ~Jar? + cand s() = ~ar Average velocity: 200% 8 = slig— i) _ [-(1/2halip + Ai? + ol = [=(2alg ~ as) + I] Ga) GA) Zar Zar ap At Tai () instantaneous velocity at = ty 106. c= BM + 630 dc__ 1008000 , ao +8 (351) ~ C1350) ~ $083,095 ~ 5085 ~ ~$1.91 weno =250 6 = S10, 107. N= f(p) (@) £°(.419) iste rate of change of gallons of gasoline sold when the price is $1.479 per gallon, (©) /(1.479) is usually negative. As prices go up, sales go down, 108, f= xur~ 1) wot tinte ‘Since the parabola passes through (0, 1) and (1,0), ‘we have: 0,1: 1 = a(0? + 0) +e = c= 1 (1,0):0 = a(t)? + (1) + 1 9 B ‘Thus, y = ax? + (a ~ 1)x + 1. From the tangent line y= x — 1, we know thatthe derivative is 1 atthe point (0. ‘The x-intercept is (2a, 0). ym 2axt (-a~1) 1 =2a(1) + (-a-1) I=a-1 ‘The area of the triangle is A 1 =-5+2 rhe net (02) Section 2.2 _ Basie Differentiation Rules and Rates of Change _119 AM, y= a= 9% yaat—9 ‘Tangent ins through (1, ~9): y+9= Gx 9-1) (@-99) +9 “The point of tangeey ae (0,0) ad (9, ~¥)- At (0,0) the slope isy(0) = ~9. At #8), the spe is») = -2 ‘Tangent lines: y~O= 9x —0) and yt He 9) -9x york-# x+y=0 9x + 4y +27=0 ma y= yam (@) Tangent lines through (0, 2xle- 0) 2 y tV—a=x “The pont of tangeny are (-/=a, ~a). At (Va, —a) the lope is y( Va) = 2/=a. ae slope is y"(~-V=a) Tangent lines: y + a = 2J=ale- J=a) andy + a= -2V=ale + V=a) ye2vraxta ye -2/-axta ‘a must be negative. Restrctio (©) Tangent Lines through (a 0): y= 0= 2xx a) w= 2st Jax = x2 ~ Jar = afr ~ 20) ‘The points oftangency are (0, 0) and (2a, 4a’). At (0, 0), the slope is yO) = 0. At (2a, 4a") the slope is y"2a) = a y= 0= 0x0) andy ~ 4a? = dale ~ 22) ‘Tangent lines: y= dx ~ 4a yao Restriction: None, «can be any real number. 120__Chapter 2__ Differentiation ws (, 1 ‘fmust be continuous at x = 2to be differentiable at ‘lim #03) = im ax? = 8a | Ba= 44d 8 ig. f0) = Jin (2+) = 440) gaan fax, x <2 ro-G It For fto be differentiable at x = 2, the left derivative must equal te right derivative. Say = 2(2) 11S, f(2) = |sin.x| is differentiable for all x # nz, ae eee 1) ihe . eso ey ity ig cn ro [oa rsd Soames Answer, 2 = 0, = 1 116. Let f(x) = cos x. = jim L@ +4) - se) £0) = i ae c08:x08 Ax — sin vsin Ax ~ cos x a r a, ED noo SP) = 0- sin x(1) ‘sinx Section 2.3 Product and Quotient Rules and Higher-Order Derivatives 1. eG) = G2 + Nee ~ 28) 2, £0) = 6x + 5)? = 2) G2 + 1Nax ~ 2) + Ge ~ 20/08) S0) = (x + 5)(388) + (8 - 2¥6) 2 ae on 2428 = At = 18e + 15s + 6-12 mae = Ge? 4 2x2 = 2s + 158-12 3M) = Hie +4) = E44) A glo) = Ji = 29) = 214-24 Wi) = HBR + (2+ ree £16) = 81-29) + (4 = shee wo Bed yn da# m2 Sap 2+ Se +s se “oar Section 2.3 5. fx) = cos x FG) = (—sin x) + 008 13x) 2.10347 G2 + 100) = x24) fen +P 9m) SE FT _ Wt Dh = a) WG) = aP w+ (9x2) See + 1) ~_ tt “aa a a) = Product and Quotient Rules and Higher-Order Derivatives 121 6 ely) = JEsinx VE cosx + £10) = Veeonx + sina Pe? & Om aa7 _ r= 12s) — (2 + 22) _ 2 = 141 ~ 4 £0 OF “ar a-F 2. 1) = St — 2%(cos x) = sin x(2x) _ xcos.x — 2 sinx 0 _ Hsin 1) — cos 344) _ _tsint + 3cost £0) a ro= a anne 14. f(a) = (2 — 2x + De = 13, £0) = (2 ~ 32x? + 3 +5) FC) = (2 ~ 34x + 3) + Qx? + 3 + 9)GP~ 3) Ox + 12x — 3x? — 18-15 15 11 fle) = xc0sx FC) = (sin 2) + (C08 2)(1) = cos x xsin x 1-8-3 ) Be ’ PG) = (2 = 2x + Gx) + (2 - 2x ~ 2) = 3x4Ge— 1 + 2G 1A ++ 1) = Ge AGH? + 2x +2) Fao 16. fa) = 122 Chapter 2 _ Differentiation 18 fl) = ro = leo) = ina ¥ (n/6)( V3/2) ~ (1/2) 77/36 Svan 18 3{ Vin 6) ‘Eunction ‘Rewrite _ Differentiate. ‘Simplify vy, y= 222 yetoek yoke? ya -32-3 = 19 a Se errs 26. 69 = PA? _ = NE2 = 29) - G - 2-2) = @= GH + 3) - O° + 3x + 20%) se = aa Se) = aa dvdr +2_ 20-0 =F Gay att feet 2. 10) =1- [Sh] e+3)4- ax) _ fet) - 6-1 SOO UF roe _G2+6+9)— 12 . es? etal lea 2460-3 Section 2.3 Product and Quotient Rules and Higher-Order Derivatives 123 29. sla) = ae =2rh 458 se) = 3. f(s) = YJ +3) = PE? +3) Alternate solution: Ste) = Ve(Ve +3) 22) gmreaften) en roy =Seueeee sa "get BR 32. ha) = G2 = 1 = 48 —2) WG) = 4x? ~ de = 4G? ~ 1) aa poy DM B= 5-3“ aeo3) Be (2-2 — Os = Ne 3) _ 2 se) = a aut —20+3 FG 3F 3 ats) at (e+ 12 Gripe (x2 + 2x +) GaP we 0) 35. fs) = (x + 4a)le ~ SNe +1) FG) = Ox? + le Sex +1) + BE + 4) Ge + 1) + BP + 43) ~ 5) (x? + 4)(a? — dx — 5) + 3x4 + xP + Ax? de + Set 1504 + x? ~ 208 4 — 36x ~ La? — 16x — 20 + Gx = 12x? + Bx? — 16 fe ISe4 ~ 48x — 33x? ~ 32x ~ 20 36. $6) = G2 = ar + Doe Hx + FO) = Qe NG? + G? +a +1) + G2 = NANG +x + 1) 4 (= Do? + NRE +1) = (2x — 1Ylet + x3 + Dx? + x + 1) + (x? — 22x? + Dx? + Dx) + (x? — a)Qa + x? + 2x + 1) aS txt ae ex 1 Had 2 OS oat 8 + ax — Bat 1 124 Chapter 2__ Differentiation 3. so) =34S ean) - + 2103) oF ae “ea F 39. f= Pint 10 = Poost + 2esint = t(toose + 2 sind) sin t+ c08 = 48. fle) = ~x + tanx FP) = 1 + sect = tats 45, g(t) = i+ Bsecr= 1! + 8 seer 20) = Fr + Bsccrtint = Fhe + Bsc ran aU = sing) _ 3, ar, y= Mast) — Heer — tans) y= Secextans ~ sec?s) = 3 seextunx ~ seex) 3 ; = p(sec x tan.x = tan? x ~ 1) #9. y= —csex~ sinx y's ese xcotx ~ cosx = cos x(esc? x — 1) = cos cot? x Si. f() = x?tanx FG) = x2 sect x + Qetan x = ax sec?.x + 2 tan x) SR y= 2esinx + x2 cox 2xcosx + 2sinx + x*(-sinx) + 2xeosx = 4xcos.x + 2sinx ~ x?sinx 38. 6) = SSS 22+ x9=28) — (C= 2708) Se) = Gas 40. (0) = (0 + 1) cos 0 F( = (G+ 1N(-sin 8) + (cos 61) = cos 6 = (0+ 1) sin 8 a2. fn) = + ootx eset = cot? = leses WWesescots sin x + cosx = xcosx + sinx ~ sinx = xeosx 82. fla) = sin xeosx FC) = sin.(~sin x) + €08.x(c08 x) = cos 2x 54, H) = 50sec 8+ Oran 6 10) = 50sec Otan 0 + Ssee 0+ Osec? 0+ tan Section 2.3, Product and Quotient Rules and Higher-Order Derivatives 128 6. 90) = (254s) +x) toe +2 110) = 22 PSEA om of answer may vary) 58, 7() = So cor sin 0 + Ocos @ -1 EERE AERO mf ansvermay vary) (0) = ore o-1 cor oF (Form of answer may vary.) T= owx y pra (= eseai(-ese roots) = + ce se reots) _ —2esexootx (= eeaP fn) 203) (QF p(y = Hocetan ) = (Soe MH) _ se tan = 1 2 2 sec (ntan 3 = 1) 1 62 fle) = sinx(sinx + 0082) 6. @ fe) = (8 ~ 3r+ DOe+2), (3) 1G) = sin (e083 ~ sins) + (sin + 6082) 08x 2 — 3x4 IU) + +2)G%-3) 4e +62 - 6x- 5 1; Slope at (1, ~3) Ie ay= = sin.xeos.x~ sin? x + sinxcos x + cbs?x = sin 2x + cos 2e f= x = sin + cos F = 1 64. (a) fla) =~ 1? = 2), (0,2) FC) = (& = (22) +(e? = 2)) = at = 2-2 ‘poy = = BOD =a) ‘flO = ~2; Slope a (0,2) c- G- ‘Tangent line: y — 2 = —2x=9y = -2x +2 £Q)= =1 15, Slope at (2,2) ° t l ‘Tangent line: y~ 2 = —I(x-2)=oy=~x+4 [ 5 o 126 Chapter 2__ Differentiation 6 @ fo) =*St, G3) 67. @ 6) = tans, @+ 0) == 0) 2 se) = f= wr £0) = 3, stope at (2, ‘Tangent line: y — o f @.@ s)= wex (32) ® ‘ ae UW #°(2) =2V%: store « (3.2) Eo “Tangent line yo2=2vilr- 2) 6V3x ~ 3y + 6- 2/3 =0 @ W-s85 @D ro eae = 16 wre Oa 1) = 716) LO" Gap 3 1 yri=-}6-9 panier dy +x-4=0 ax (4 meat me w= tg + 16)(16) (2x) _ 256-16? fg) = G2 + 04) = Ax(2x) __24— ax? £6) = SH ee 8 ie G+ oF wre 0) = 4=15 2 1” 25 2,16 yaa t os 2Sy ~ 2x 160 25y— 2x + 16=0 Section 2,3 Product and Quotient Rules and Higher-Order Derivatives 127 75, 0) = 25? _ (r= Ney - 20) _ (2+ Ne» ~ V0) 2) = x = 2129) £0) = E=Nea ste) a re ea winds ent - Mirth “G@- +P Fle) = O whens = Dore = 2. £0) = Owhen x= 0. ‘Horizontal tangents are at (0,0) ‘Horizontal tangent is at (0, 0). and (2, 4). F'G) = O for 4 — 4x=0 => x= 1 sy =2 {has a horizontal tangent a (1,2). 16, 4) = 54 gy x 2D) =be= 806) 7 oe et T_ Me) re =F GF e-7 - @-F £6) = Ofer x= 1,7 04, (=H {has horizontal tangent m. 0) == 78. fs) = voy =O) oe FO GaP 10" Ga iF “G=F Let (x,y) = (2/(x~ 1)) bea ltt gro G- Det) (ar — he 4c Or + 40 (- 2@e- 1b) = -1.90)=2.7(3) = 40) = -1 ‘Two tangent lines yriendent) ae ye tee yr2=-1e-2) = yanxtd 128 __ Chapter 2 Differentiation +25 - 5x4 40) 6 a en wR Srt4__ ae, 2044 3) = GD “erp &+2)~ =f) +2 Sand g differ by a constant 81, @ p'G) =F’ Ge) + F086) Pr) =S'alt) +704") = 10) + 6 ~ Gy) = AL = L008" 6 (4) = AL) = fe) oP a = 2D=1O 1 83, Area = A() = (2t + 1I)Vi = 287 + 8? Bara) 4 bps ce ea cwnelen a@ ott awit ofS + rig) tes BoP + ay) cemP/sce 88, = 10; £2 - (@ When x = 10: € = ~$38.13, ac () Whenx = 15: = $1037 (© Wiens =20 = 3580 As the order size increases, the cost per item decreases. 87. PO) = sr +34 ai (50 + AN) — (029 ro = sof =e = O57 - am sos] P’Q) ~ 31.55 bacteria per hour 0. pq) = Moots 3) = ain = 3040) _ a) = MOOD) = (in #200) | eeour= ens sing + 2x _ sing ~ 3x4 Se a0) = =f) +5 F and g differ by a constant 82. (@) p'(s) = f'Gdels) + fle)e') a 1 = Pp) 768) + 10) = fy) = RONG) = fae") oro a Madd 2 ot M2 ACD 23 #4. v= arth= att +2(5Vi) = Hors ae a2 Je S42 a eter k a. p= t wk awe 88. F ae 2Gmm, a7 F@ @ Section 2.3 Product and Quotient Rules and Higher-Order Derivatives 129 42] = {sin (0) ~ (1(c0s 2) (sina? feta). snscsn a= (on Neo sin (sina a cot] = 90, fx) = sex 86) = esex, (0,27) Le) =e) Lanz teens! 3a Je AES 91. (a) nf?) 3.580619 + 82.5771? — 603.601 + 1667.5 0) 0, 2 ¥(0) = 0.136119 + 3.16512 — 23.02 + 59.8 Ros] Re MO) 0.13618 + 3.16512 ~ 23.021 + 59.8 % @A= i) ~ =338060 + 82.5770 A a . (© A'@ represents the rate of change ofthe average retail ae value per 1000 motor homes. A represents the average retail value (in millions of dollars) per 1000 motor homes. 92. (a) sin = (&) (8) = r{—cse 8 + cot 6) rth=reed woo) = »(2) cae aca = =3960(2 - V3) = -7920 V3 mifradian 2 93, 7) = 402 94 soy 9 = 62 poni- FQ) = 302 ro-2 130__Chaprer 2__ Differentiation 95. fle) = 0) = 2) 10 GF I= ce 97. fle) = 3sinx F/G) = 3c0sx $°G) = ~3sinx 99. $'0) 100, f*(3) = 2-204 se) 2) =0 One such function is f(x) = (x = 2). 108. f(x) = 2e(x) + Ma) S(0) = 292) + 4) F(2) = 292) + WO) 2) +4 ° =) sor. 70) = 48 = Mae's) — e(h's) EK) DOF = W2)g'2) = g(2)h'Q2) £0) HOF 2 EE) = 394) Cy =-10 Bator xta-2 96. fa) = sen1ed s--3 98. f(s) = secx Ft) = seextanx F7*(8) = see x{sec* x) + tan x{(sec x tan x) = sec x(sectx + tant x) 101, #"6) = 2 102, 49) = 28 +1 F949) = Hoe la) =2 0) = 0 4104, The graph of a differentiable function fsuch that f > 0 and f’ < 0 for all eal numbers x would, in general, look like the graph below. 106. f(x) = 4 ~ hx) Ft) = =H) LQ) = -W'Q)= -4 108, /69) = gG)460) FO) = eC)WG) + He’) SO) = g(@W'2) + W2}e2) NA) + (19-2) =4 Section 2.3 Product and Quotient Rules and Higher-Order Derivatives 131 110, It appears that fis cubic; so f” would be quadratic and Tt appears that fis quadratic; so f" would be linear and f* SF" would be linear. would be constant. a. 12, may MS, i) = 36-7, 0056 7 (i) = 28 toy r ¥(3) = 27 m/sec a3) ‘The speed of the object is decreasing. m/sec 116. 4) ©) a9) = aay ap = A RM ay <2 154109 = tconey ©) 00) = Baye TSR 2A ae (© lao) = aa gp 05 Wise 117, i) = 8251 + 66 Average velocity on: v0) = ~16501 + 66 topnn5™25= 2. gr a() = -1650 ° 99 - 57 ieee) oT] if[2]J3 ]4 trays SP = as. ) ) 0 |_s7as|_99_| i375 | 152 pasos) v(e) = 50) (ft/sec) 6 49.5 33, 16.5 0 (2, 3)is 3-2 7247S. ai = v0 (t/see) | 165 | ~165 [165 | ~165 | -165 - (5aig B2=H2EIS gas a3 132_Chapter2__ Differentiation me; (b) The price peds up (accelerates) when @ > O and : slows dow when a-< 0. hh ‘Answers will vary, + peston function » veloc funtion «secelerton fnction 0, so) =4 non — 1) — 2)-+-(2K{1) = nt = 1)%n)in = 1)(n— 2) == (2)(0) = rh = 1) 3-2 + 1 (ead “nfatorat”) 1) = aa 121. fle) = eGHG) (FO) = sth‘) + hina’) FG) = alah") + gH) + Male") + HWE) = AGING) + 26 GH’) + Hae") F7G) = gd") + gNL) + 2e WIND) + 2e"C0IN') + Hale) + Hee") = adh") + 3e°ODK") + 3e"CHE) + eM) FOUR) = ala)niM(x) + gad" G) + 3a IAM) + 30°C) + 3e"O)KG) + 3e"G)K'O) + a (a)n'@) + gE) = ela) + Ae QING) + GETHIN) + de") + CHA) nn — 1)(n - 2+ -Q(1) = nln — 1) 190) = 8M + Tg ena) ONDE OM) * B= er ou) aaron n(n — 1)(n = 2)- - - (21) (yn) an BRM = 3H a) - ype" + nin in -2)-- Ql) g-y ete “Bae names = a) +g MM) + are retreat + + aa + eh GAG) Notes n! = nn 1) + °3+2 1 (read “n factorial”) xf") + SU) + FG) = a") + 2F°G) [xf] = x7) + £6) + 27°0) = af") + 37°) In general, [xfCx)}® = xf) + ngf*-MG). Section 2.3, 123, f) = xt sinx SG) = x" 008 x + mx"! sin x \(veos.x + msina) When n = 1: #"@) = xcosx + sin When n = 2: (2) = a{xcos x + 2sinx) When n = 3: (2) = 32(cc0sx + 3 sin) When n = 4: #6) = %xc0s x + Asin) For general n, (x) 3(xcos x + sin). cama flex wy" + Dey! = 2sinx +3 y= 2eosx 2sinx Product and Quotient Rules and Higher-Order Derivatives 133 asin x — met! cos x = =x fe sin x + me0sx) ~ taba none Wien = 1:6) = —#8O When n = 2: f(a) = ~2SRE LACES When n = 3: £13) = When n = & fe) = 126, ya 28-6410 y= 62-6 y= ie yr=2 ay" = 2y' = 12 — a{128) — 2627 ~ 6 128, y= Scosx + sine yim ~Bsing + cosx y= Boos ~ sinx Yt 7m — Rains + Qeine + 3) y+ y= (Beosx— sins) + (Bcosx + sins) = 0 129, False. fy = feel), then 130, True. isa fourth-degree 131. The polynomial. . Da sodas) + e600. 2 - ownenn > 4 10) = 1080) + ef) oF = F100) + 51010) =0 132, Tue 133, Tre 134 Tre, 135, fly) = ax? + be te S'@) = a+b intercept at (1,0: O= a+ b+ 2. 7on graph: 7 = 4a + 2b +6 Sipe 10 at (2,7: Subtracting the third equation from the second, ‘Then, 10 = 4(3) +b =e b= =2. Finally, ~ fla) = 3x2 = 2e= 1 fab Ino() = cthen a(t) =v") = 0. b+ e. Subsracting this equation from the first, 3 = a (-2)+c= c= =I. 134 Chapter 2__ Differentiation 136. fle) = ax + bx? + ox + da 40 £'G) = 3ax? + 2bx +e 2b « JB Tae £)=0 =x = (@) No horizontal tangents: f"(x) #0 (b) Exactly one horizontal tangent (©) Exactly two horizontal tangents 47 — Vac <0 4 — 12ac = 0 4? — 12ac > 0 Example: a= ¢= 1, Example: @ = 1,b = 3,6" 3: Example: b= 1a = 1e= 0: fa)=P +x f(x) = 28 + 3x? + Be fQ)=8 +2 femusea 138. @) Ue! —S)" =a" +f’ Se" S's i =f-fe Tne 2x, ite 0) te) = ess ifx< 4 =| (©) (ie)"= Ue’ +8)" no) a [% ifx>0 era se) { i eC =f t Oe tis £7() does not exist since the left and right derivatives a ee ‘are not equal. Section 2.4 The Chain Rule Ly = (6 - 5}* um 6r-5 yew ayes weet een Bye VET wees yeu 4. y = 3 tan(x’) un me y= 3tanu ‘S. y= cschx wm cscx yee 6 y-cat ve foewe % y= (Qe- 7 & y= 34 x5 y= 3Qx — 12) = 62x ~ 7)? y= 15(4 — x2)4(—28) = ~30x(4 ~ 2°) 9 g(x) = 3(4 — 93) 10. f() = (91 + 29° 8°) = 12(4 = 9)4-9) CD 110 = 201+ 29-10) = yeh Section 2.4 The Chain Rule _135 ML. f= = 9 12, 46s) = JE= Be - 39)! PO=Y0-AE = 5 1) = Ys — 3x)-¥(-3) = 81) = 5 - 8"-3) = ee rr 1, y= Or + 498 14. et) = VETBFT = VEAP = fk y= 708 + 490089) = Gs sof, 221 18, y= 24-294 16. #0) = ~39T= 5 = 302-99 ye (je = 34-29) F(a) = ~3(2 ~ 9x) £00 = Yo 9994-9) = Toa Wn y=@-2)7 8 = 3H 19. f= (- 3)? yim 1-2-0) = si) = (2431-1 P= 0-3) = G5 GF a) = — M2 + 34 1-70 +3) 7 a y= - 35 we y= Ger 2He 2 d= Jats ym 30043 Ba Hes ayn a) =(2 = 28 1s la 9) Ise + 3-4 = 1 (= 5 ~ 2-372) GF ET goed 23. fx) = Hx ~ 2) 2A, fla) = x(x — 9P [4x — 22] + (= 22x) SG) = {33x — 9P(3)] + Gx — 91) = Dex = 2)°2e + ~ 2) = Gx — 99x + 3x 9] = 2x(x — 2P°Gx — 2) = 21x — 3P(4x — 3) 25, yaxJT> Fa al 3)" 5. eae do -2-7ea9] + 0-2-0) et -aes-ae 02H 2) Lo 28 2 7xGx = 32) 2Wie=# = 97-29) + 16 = 24 +x 136 __Chapter2__ Differentiation 2 y= Tae yaa -He + 900] + Ge + I-A) = G2 + 19 + IIA B+ Ae + (e+ D] 1 “wtp (2 +2) — x + 96 (+ ar a 2G + 52 — 10 ~ 24) (ao a s=(152) TG t+ 4920) = het + 4)-12(4x) ed tt 4— 2x4 aot “ara 30. no= (5) w= A525) ye +22) = AG Pe, (e+e 22 9) _ 28 = 8) @ + ery 2 (ea) a + y(-2) = (1 = 29) sx? — 2)"((ax + 3)6x) ~ (3x2 ~ 2)0) 10) roa) ee) = ap = 3G? — 26x? + 18x + 4) ary - Ort 3 _ (Se — 27? + 9x + 2) - Grae Ji+1 2 Boye aT t Moe VaaT T a lege 4s? “HES 1 YT Tae + 1 F Vie + 1p? The zero of y’comesponds to Gr 1" has no 2208. a the point onthe graph of y ‘where the tangent ln is horizontal as y= fet 7 ell IE . L yo nee 4 7—| (O° TRST aT bas no zero + a T Section2.4 The Chain Rule 137 a He = vermeayempae Eo # the points on the graph of y ‘where the tangent lines are horizontal. “The zeros of y’ correspond tothe points on the graph of y where the tangent ines are horizontal. Bye sing y= sin2e y=sn(3) y= 008 x y! = 2008 2x va ("eeal® y@=1 yO) =2 yO =3 ” 5) oa(5) 1 eyele in [0, 22] 2 eyees in [0,2] ‘3 eyeles in [0,20] ont ‘The slope of sin ax at the ” 2 ‘origins a. Half cycle in (0, 27] ‘The slope of sin ax atthe origin it a, 41. y= cos de 4, y= singe 48, ols) = 3tan de 44 Ws) = sels?) a ©. scos ax a) = 12sec te WG) = 2x sects?) ants) B= -3sinae 2 45, y = sina? = sin(ax?) 46, y = cos(l — 24)? = cos((t ~ 28}) sin(1 ~ 2x)°@2(1 ~ 23)(-2)) = 4(1 ~ 28) sin(t = 23) y= cos(n2x2)[2r2x] = 2a costar’) = sel) tan(3¢) (40) sec>(4o)b + tan(t) sec($0) tanh) = }see(dofsee*(de) + ano] 41, Wa) = sin 2xe08 2 48. ‘1 (x) = sin 2x(2 sin 2x) + cos 212008 2x) = 2cos? 2x ~ 2 sin? 2x = 2eos 4x ‘Alternate solution: H(2) = }sin 4x h’G) = feos 4x(4) = 2.008 4x 50, s(e) = Soyo cosy sin |) = cos v(cos ») + sin o(—sin») = cos?» ~ sin?v = cos 2v cos? wt = Slo a)? 53. (0) = }sin?20 = Mein 207 10 cos art(—sin ai)(n) £(@) = 2Az)(sin 26) (cos 20)(2) — 10m sin m2)(c08 =) in 20.0820 = tain 40 ~Smsin2at SI y= 4sectx = Bec see xtanx = Bec? xtan 138_Chapter2__ Differentiation S40) = 2eot(ar + 2) WI) = 4 cot(at + 2)(—esc%(at + 2)(a)) = —Aarcot(art + 2) esc%(at + 2) 56. y= 3x Seostms)? = 3x ~ S costars?) x. Bina + Ssin(a%x?V2n%) = 3+ 10m sin(aa)? 58. ym sina + (sins)? y= cosxt9($228) + Lisina) 2 con 60. y= Gx + 49)", 0,2) y= HG8 + 4) OH +4) bets “Ge + aye y= @. fe) = Get 304 (438) 1G) = -262 ~ 3872" ~ 3) = LO=-% at 2 ey = = DW = +O) Ox 3F 64. fe) = 2,3) LQ)=-5 3 secX(a ~ 1) 55. 0) £0 = 6 see(at ~ 1) seals ~ 1) tan(at ~ 1m) = Gr sent t) anne — 1) = Sesilae—) (rt — 1) st. y= Je+ t sin(au? = Vi +f sin(te?) Boley ld Bm he +t cos(ae(te) = os zea? "TAt™ (2x) 59. 0) = (2 + 26+ 9), (2,4) (= FO +24 + 8)-PQe + 2) a. f= Aga see 4), £10) = ~340 ~ 4)>G04) “--2 sen=-Z © 0-242, 0-2) (jy = = DG) = Gr + 2)(1) ro -e £0) = -5 37 = see%(23), (0,36) ~3 sec*(2x)[2 sec(2x) tan(2x)) ~6 sec'2x) tan(2x) y@)=0 YF feos (/2) is undefined. ©. @ fe) = JE=F, 6.5) 1 F') = 302 = 216) \ eo) 1g) =2 £O)=3 +. ‘Tangent ln: 68. @) fo) = PF FS, 02,2) © (e+ oo sherse “5. wae +5 +3 B 10) = 365 +50 =F ‘Tangent line: y~ 2 = Boe ~ 2) =» 13x 9y- 8 =0 ©@ y= P40, C10) 70. @ fe) == x79, (1.4) 9s the + 1K6e) SAS + 1) $6) = 20 — 8-19-29) = y(-1) = 12-1) 12 , 4 2 ‘Tangent line: y ~ 1 = ~12(x + 1) 1) = 5g 3 ys ‘Tangent line: y — 4 © “ uo + | " ad) 71. (fla) = sin2x, (7,0) 2 awe KA. fm) =2 Y — Ly y= 2e- 2) =o 2e-y-29=0 Section 2.4 The Chain Rule _139 ee 3o- =F 140__Chapter 2_ Differentiation Ty eos3e (¢ -#) © 74, (@) y= Dean? x, y= Otantx sect »(2)= 602) = 12 apeine 9-2 1fe~2) y=ix+2-3m o 16. (a) fa) = VeQ-3), (4,8) 2(Sx— 2) ave 73. @) 46) = 1s, (3.1) yotedeZ)asae-y 402) =0 © 4 AAU Gs) 3e “Vera ae? +342) @ra- 75.(@) al v= mi) = SEE ( 1g -z2EEEy 221 O° aT 8.() y= (VET, (2-10) SP 48-9 OWT 10-2 yOuF =2q- (b) y + 10= af 2) 221, 74 2 a 2 ste) nify) = t= WM? = x(2e S)= =? Gi poh “GF Horizontal tangent at (1 1) Section 2.4 The Chain Rule _141 1. f0)= JB=F, @,4) £0) = Fs yeh 2S, tg tie a1. Sls) = 2oosx + sin2x, O< x < 2e £0) 2sinx + 2.08 2 2sinx +2 —4sintx = 0 2sintx + sinx—1=0 (sinx + N@sinx~ 1) =0 Horizontal tangents at x = Hz anger athe pons ‘sm _3v3 wy 83. fle) = 262 - 1) £'G) = 63? ~ 1722) = 1ax(et ~ 282 + 1) = 12s - 24 + 12x #70) = 60x4 — Te +12 = 12(S2* — 1)? 1) 1&2 Chapter 2__ Differentiation 4 fe) = - 2) " y-2 =i S0) =~ 2 £0) = 26-2) = G2 86, f(9) = sec! mx 1G) = 280 mae see astan 13) = 2ersec? mxtan mx 85. f60 SG) = Beeosx? 7G) = 2afae—sin 2] + 2.00832 = feos x8 — 23 sin] IG) = 2ersec® rx(sec? 3)(z) + Dertan a(2a sec? axtan 3) = 2m? sect mx + 4a? sec? ox tan? me = 2m? sect ma(sec? wx + 2a? ws) = 2a? sect ax(3 sec? wx ~ 2) s. Ha) =3ar+ 0, (1.5) WG) = Bx + 1G) = Ge + DF W) = 23x + 1)) = 63x + 1) we) = 24 89. fle) = costs’), (0,1) $'Gs) = ~sin(x?)(2x) = ~2x sin(x%) $C) = ~2xcos(x")(2x) ~ 2sin(x?) = — 4x? cos(x?) — 2 sin(x?) sO =0 ‘The zeros off" correspond to the points where the graph of fbas horizontal tangents. rar, (0 fesanr £0) = Yer arma, 90. g(t) = tan(2r, 2° = 280022) 8°() = 4se0(20) + seo(2e tan(2e)2 = 8 secH(21)tan(2) = 323 ‘is decreasing on (—co, ~1) s0,f’ must be negative there. ‘is increasing on (1, 20) so f” must be positive there. Section 2.4 The Chain Rule __143 “The zeros off" correspond to the points where the graph “The zeros of corespodto the pints where he graph of fas horizontal tangents. of fas horizontal tangents. 95. 66) = s(38) 96, 90) =f) £'@) = F189) = 9’) = 3/69) 80) =) = 0G) = 2°67) 97. (a) 10) = ebohls) (ae) FG) = 8G)A'G) + 9°) HO) “(ata))n') $15) = (3-2) + (618) = 24 $15) = 63-2) = ~26') o oe) Need g/(3) to find (5). a © fe) = (eo? 1g) = Heda") = a6) 0°) “(e) = alate“) re) hae £'G0) = Slab’) ve), QO — £6) = 3(-3%6) = 162 f= BF 98, () s(0) = 0) -2=9 4/0) =s'0) (b) Als) = 2f(2) => HQ) = 24°) mee E21 ORE e ee © 10) = f-3) = 0) = CBN) 5-39-39) fe) | 4 | § | -¥]-1 | -2 | -4 ence, you ned wo know f"(~32). e@] +] [4l-1[-2[-« woe | [-4[-2[-4|-8 (C1) = ~37'@) = (3-4) = 12 r'G) pt © 0) =f +298) = +2) se) |-$ | -1| -2] -4 Hence, you need to know f'(x + 2). #(-2) = $0) = ~hete 1, h’@) = f'@Q)2") 99. (@) hs) = Fle), ot) = 4 @'() = -h FO = w(t) = F(G(0)2"(1) “@e') 1-4) =4 = 0), 165) = 6, #6) “(0))4'0) $6) = (FBI'S) = #1) ‘Since g’(6) does not exist, s’(5) is not defined. 1,(6) does not exist. 144__Chaper 2 Differentiation (eta) S'(G)e") £168") “5)() 201. (a) F = 132,400(331 ~ vo! = (~1)(132,400)331 = )-*(—1) 132,400 Br = When v = 30, F’= 1.46. wa y= Leo tar— ben = y= 3 12sin 123] ~ Hf12 00s 129 ~Asin 12¢— 3.0512 When = 1/8, y= 025 feet and y= 4 fet per second. 104, y = A.cos or 3s aS = 1.95 75.08 ot (@) Amplitude: A y tn on Period: 10=9 o = 2% = ea y= 175 cos T() = 65 + 21s ©) ©) = ss) 5) = aFO) SO) $0) = e(FO)F'O) = 8'@)2) = (-D@) (&) F = 132,400(331 + ¥)~! Fim (~1)(132,400)6331 + ¥)-™(1) = 132.400 Bry When v = 30, F”= =1.016, 103. 9 = 02 cos 81 ‘The maximum angular displacements 6 = 02 (since =1 Seos8"s 1) @ ~ oaf-8 sin] = ~1.6sin8¢ When t = 3, d0/di = ~1.6 sin 24 ~ 1.4489 radians per second. 108, $= C(R? = r2) as Rae a” eG -* 3) Since ris constant, we have dr/dt = 0 and as ‘a (1.76 x 10°)(2)(1.2 « 10-310") = 4204 x 10-7 = 0.04224, © TO ~11e0 (@) The temperature changes most rapidly in the spring (April-June) and fall (September-November). 107, (a) x= = 1.63728 + 19:3120# ~ 0.50821 ~ 0.6162 ©) C= 60x + 1350 Section 2.4 The Chain Rule 148 = 60(-1.63728 + 19:31208? ~ 0.5082 ~ 0.6162) + 1350 £ = 60(—491162 + 38.6241 — 0.5082) = 294696 + 2317.41 ~ 30.492 ‘The function dC/ad is quadkati, not linear. The cost function levels off atthe end of the day, perhaps due to fatigue. 108, f(x) = sin x (@) £0) = eos Bx 1G) = ~BFsin Bx I"(a) = — BP cos Bx f= Btsin Bx () £°G) + B/C) = — PP sin Bx + pPlsin Bx) = 0 (2%) = (-1) 6% sin Be F-03) = (1B cos Bx 110. (@) F(a) = f'G))e") ri) = Fe") [Note that (1) = 4.and“(#) = ‘Also, g’(1) = 0. Thus, r’(1) = 0. x + cos?x= I= e'() <0 = 2sinxcosx + 2c0s.(-sinx) = 0 112, @its(-2) =f), then 4 4 Ata = St-s00) £1 = -F'@) a) = f°). ‘Thus, f(x) is even. m3, [a= Vit gle = $Lve] = 308-72 , u#0 109. fx + p) = fl) for all x. (@) Yes, f'Ce + p) = JC), which shows tha jis periodic as well. () Yes, let 2) = £29), 80 8°) Since fis periodic, so is 2f'Qx). © s'@) = s'FO)F'O) sO = 2ST) roomast=Sr(6 19 =S. rm () tant + 1 = sect sQ) +1 =f) ‘Taking derivatives of both sides, g’(x) = f’(x). guivalendly, 0) = 2see x seex tnx and Gs) = Dian sec, which ae the same, © Ws-3) = 70), then Sua) - 2u0) zl ze SMD =F) 114, Gt) = [2x — 3] #0) = (7 2-3 146__ Chapter 2__ Differentiation us. /6) = | ~ 4 116. fs) = [a cosx 117. f(s) = [sins] rey=2( = x#a2 we ins + feos, #0 FG) = e082), a 4 be U8. (@) f(a) = tan f= sect £0) = FQ) =F F9)= Feet tan (9) r= Fa PAG) = Fe — 1) +) = Fe 1) +1 pe) = {EZ )a— + roy 9 +A0)= Foe P+ Fe— +1 2 (©) Py isa better approximation than P,. (@) The aceuracy worsens as you move away from x = 119. (a) fs) = seo(2s) ® + $°G) = (see 29)(tan 2s) KJ 40) = 2f2(0e 25am 2x) tan 2+ 260 25) s0e*25)(2) K2Z os (see 2x)tan? 2x) + sec? 2x] Ag)=se(3)=2 (© Pisa tee aroxinaton an, (2) 2002) em2) = 405 (© Me wry sone asso now aay £(2)- 420) +2156 Pi =aVi(x- 2) +2 P46) = $060(x- 2) + 45 nef nald = 9), then 121, False. If f(a) = sin? 2s, then 122. Trve a1). FG) = 2(sin 28)(2.c0s 22) Section 2.5 _ Implicit Differentiation _147 13. $0) = 0, sin + ay sind +++ + ay ine F'(3) = a, c08 x + 2a cos 2x + +++ + na, cos mx I" Joy + 20, +--+ + ma = (70) fla) - f00) x0 yt 2a Ho nay 7 (et = 1)" 14/6) = PAG)n + Ife — 1prkct me = GER DACs) — (0 + tke) GF P,G) = (= []= G4 = neve = + nk) 2, = ey = 0 =~ Ato, 260) = 1. ‘We now use mathematical induction to verify that P,(1) = (~#)%n! for n = 0. Assume true forn. Then Pyui(l) = (2 + DEP) Section 2.5 Implicit Differentiation Lt +y=36 2 Bo 16 & xi + yi 2x + y= 0 2x — By'=0 Lema bynyao a > 148 Chapter 2__ Differentiation 6 xy tyre By! + Day +»? + Ing’ = 0 2 x0 #23) 8 Gy)? =x + 2y=0 eo)!’ + 9) — 1+ 29" = 0 ppt ppc ityrso ay’ + y— Day + Ay’ = 0 yn Baz Ada +x 1. dsinvomy #1 ‘2{sin x(—sin y)y’ + cos y(cos x)] = 0 _ o08xc0s y sin sin y = cotxeoty 2 (sin mx + c08 m3) sin mx + cos ry) cos ax ~ asin xy)y’ 2 xy myx 0 Beyty! + 3x4 - y= 0 (xy ~ Dy’= 1-38? yates 1S RFT a x — Bey + 2g? = 12 3x1 — Sxty’— Gay + dy’ +2? = 0 (day ~ 3339" = Gay — 302 — 297 _ w= AL, siny + 2¢082y=1 cos x ~ Asin 2y)y’= 0 cose Hsin 29 13, sing = a(1 + tany) cos.x = a(sec? yy’ + (1 + tan y)(1) sr e0s mx ~ asin myy’= 0 osx —tany — 1 oxsecty 18 = sab) 16 x= seed b+ posto) ; = Yee tant y/ xeoste)y’ = y costo) boos yiny ost) = 17. @ # + y= 16 yate— 2 yeaa e © Explicit: Lag — 2-1 2 hag - 34-29) Vie=# "2 Jie= = 2" Seep) tant) = real) eof) © @) Implicit: 2x + 2yy"= 0 Section 2.5 Implicit Differentiation 149 18, (a) GP — ar +4) + (FR + 6 +9) = -9 4449 G2 + (43% =4, Circle G+ 3P = 4-@—2P y= 3tVa= (© Explicitly: Bm alta (e— 2A A-2)e- 2) =(x- 2) eared @ Implicit: aa > 2x + dy" 4+ 6"=0 “Ta-e- (y+ 6y’= -2he~2) 19. (@ 16? = 144-92 y= fas - 98) = 206 - 2) yok iO (6) Explicitly: ® | Bag — 9-14 ae SRS )-(—2x) o a8 16 ) 4. "I, | —t- (© Explicitly (© Implicit: 99? - 7 = 9 Hx? + 9)-(2x) I8yy’— 2x = 0 p T8yy! = 2x = - ok ve PELE BED” TER)” Oy Yee mynd ay’ +x) =0 aya ny y * ALD y= At(4, 0) y’ 150__Chaprer 2__ Differentiation Be Peuya 22 + 4)@s) ~ (4 - 4) era 16 a+ _& Oo GP At (2,0): yi undefined. 2s, PP 4 yes Zeus 4 2y-anyr= geet y’=0 Atk y= 5 n. tan(s +») =x (ty) se0% +) =1 yo teecety see + 9) ase t tare) FT =~ sine +) =-32) oe eT At (0,0): y’ 2. (+ aya8 (2 + ay’ +3020 AtQth y= = (cr. you could jus sate fry: y= 85) cd G+peety B+ ary tigi t yaar ty arty + an? =0 xytar=0 BY + Day + Dy + y= 0 (2 + 2y)y’ = —(? + 20) = iy + 2x) SAD) AoA, Diy % PHP Rhy tt Bx + 3y2y/= day’ + Ay (392 = Ae)y! = Ay — 3a 4y = 3x G4) 4-28 = beaty = oy - wa 30. (= ayies AL2,25: 31 (2 + P= arty 2G? + Rx + 2yy') = Aety’ + 8) Ae + Aetyy! + dag? + dyty’ = ety’ + Bay Ay’ + yy’ = Axty! = By ~ ax? — dy? 4y'Gy +P — x4) = AQay — 2 — 7) At (It 9’ 3 (y-2F = 4-3), 40) 2y-2y" A140): 9! ‘angen line: y ~ 0 = ~1(e~ 4) yorrt4 3. an ALD: 9" ‘Tangent ie: y = 1 = ~166 ~ 1) yerxt2 a7. ay? 9x2 - 4? = 0, (-4,2V3) 22 2yy" + 2? ~ 18x — 839" At (~4,2V3} Section 2.5 _ Implicit Differentiation _151 Pty by 32 + 3y4y'— Gy’ — 6 (16/3) = (16/9) _ 32. 4 (64/9) — (8/3) 5 G+ +O-2 AtG,4) y= =2 ‘Tangent line: y a2 ~ 6V3xy + 1392-16 = 0, (V3.1) Lax — 6V3ay’ ~ 6V3y + 26yy’ wane Salt na ‘Tangent line: y - 1 = ~V3(x- V3) ~Vie+4 B+ =, a) 2 2 Deas By try Be tay’ = 0 182_ Chapter 2__ Differentiation 40. yee + y= 2%, (1,1) 2. 3G? + y®)? = 100627 - y7), (4,2) ls? + yP(Qx + 2yy') = 100(2x — 2yy") vat + yt = 2st A (4,2) dyya? + day? + Ay'y! = de Atl, De dit De hyiad 6(16 + 4)(8 + 4y’) = 100(8 ~ 4’) {960 + 480)" = 800 ~ 400y’ ‘Tangent line: (1,2) woke AL(L,2): y'= -2 ‘Tangent line: y— 2 = eee nary So ee eae eee By’ 2 a@ = 6-2 @ 5-$-1 3 B-Boy 8 z ayn Pg 7 : y 39 Se a), Tagen ie a (3) r= M4) At (3, =2): y’ 3-2 Tangent ine: 9 +2 = 2x44, Tangent line, note: Fon pu, 3 Section 2.5 _ Implicit Differentiation _153 8 uny=x “4 y'secty = 1 ~siny +y" Bey 3x ~ 4y = 0 a At (3,4): ‘Tangent line: y (e+ 3) =p ae ~ dy $25 =0 Normal tine: y~ 4= 80+ 3) =» 4r + 3y = 0 Fen Da met Vy - Bi 2 ‘Normal line: y ~ V3 (2) => V5x-2y=0 55, tye 2e + 29" 0 y lope of tangent line y 2 soe of smal ine Let (x.y) Bea point on the cic. I = 0, then the tangent ine is horizontal, the normal lin is vertical and, hence, passes through the origin. I x5 # 0, then the equation ofthe normal line is = 2e- yon Bex) % yn Be ‘Which passes through the origin, 57. 2522 + 16)? + 20x ~ 160y + 400 = 0 50x + 32yy" + 200 ~ 160y’= 0 200 + So 160~ 329 y Horizontal tangents occur when 25(16) + 162 + 200(—4) ~ 1609 + 400 ay ~ 10 Horizontal tangents: (~4,0), (4,10) ‘Vertical tangents occur when y = 5: 25:2 + 400 + 200% ~ 800 + 400 = 0 2sxlx + 8) =< Ome Vertical tangents: (0,5),(~8,5) y= 0,10 Section 2.5 _ Implicit Differentiation 155 Ex) 1at(1,2) ‘Equation of normal line at (1,2) is y ~ 2 = —1(e~ 1), ‘3 — x The centers ofthe circles must be on the ‘normal line and ata distance of 4 units from (I, 2). ‘Therefore, @-1P +(G-2) -2F = 16 2-18 = 16 xe1b2V2 ‘Centers of the circles: (1 + 22,2 - 2V2) and (1 22,2 +22) Equations: (c~ 1 - 23)? + (92+ 2V3)*= 16 (= 142V2)+ (y-2-2V2)= 16 186 Chapter 2 _ Differentiation 58. aa + yt Bet Ay H4= 0 Be + dy’ 8 + 4y"=0 Bea” ye? Horizontal tangents occur when x = 4? +9? = 81) + 4y + 4=0 w+ dy= y+ 4) = Oy = Horizontal tangents: (1, 0), (1, ~4) Vertical tangents occur when y = ~2: At + (-2)? — Be + 4(-2) +4 =0 42 ~ Bx = 4x(x ~ 2) ‘Vertical tangents: (0, ~2), (2-2) ‘59, Find the points of intersection by letting y? = 4x in the equation 23? + y* = 6, B+ dr=6 and (+I 1)=0 ‘The curves intersect at (1, +2). per 4, liga: Paraboa: te Ae + yy Dy 4 y y ‘At (1,2) the slopes are: yert yell At (1, ~2), the slopes are: yet yaod ‘Tangent are perpendicular. (60, Find the points of intersection by letting y? = 2° inthe equation 2x? + 3)? = 5, 2e+38=5 and ae +25 =0 Ree) Intersect when Points of intersection: (1,1) pox 22 +39 2yy! = 3 4e+ 6y'=0 ES y= 2 At (1,1) the slopes are: ‘Tangents are perpendicular. Section 2.5 Implicit Differentiation _157 61. y= —xandx= siny Point of intersection: (0,0) yim secy ‘At (0,0), the slopes are: yecl yet ‘Tangents are perpendicular, 62. Rewriting each equation and differentiating: #=30-1) 2 yap - je yas For each value of x, the derivatives are negative reciprocals ofeach other. Thus, the tangent lines are orthogonal at both pont of intersection. 8 oyec Boyek F * wi ty=0 2x ~ 2’ =0 VA ‘At any point of intersection (x,3) the product ofthe * * slopes is (~y/s)x/y) = ~The curves are orthogonal ‘Atte point of intersection (x,y, the product ofthe slopes is (-3/9)(8) = ("x/K3)(K) = “1. The curves are onhogonl 66, 2 — Aw + P= 10 (@) 2x ~ 39? — Gyy” + 3y4y7= 0 (Gy + 3y%)y’ = 39? — 2 3 — ae 1 Gy Hyd _ gy Vy yo 2 FG OV Gt PG HO wo $- not yBaae% x- 39) 8 = Gy - 39% 158 Chapter 2__ Differentiation 61, cos my ~ 3sin-ax (@) ~a sin(ay)y’ — 3a cos ax = 0 sin wy - 2 — 9 costm) H = eo w sin(ay) 7 30 sm) 0 = oe C4 sin() $F = 3 costa) (69. A function is in explicit form ify s writen as a function of x y= f(a) For example, y = 2. An implicit equation is notin the form y = f(a). For example, x* + y= 5, = aay? tye © ya3—9= 48 -he 36 = 162 - x4 x= 167 +36=0 pat gg 68. 4sin.xcosy = 1 (@) 4.sin.a(—sin yy’ + 4.008 x 008 y = 0 cos x cos y ”*sinx sin y ) 4sin(-siny) 2 + 4008 cosy = 0 5 nya corzcony = instny 70. Given an implicit equation, first differentiate both sides with respect tox. Collect all terms involving y’on the left, ‘and all other terms tothe right. Factor out y“on the left side. Finally divide both sides by the left-hand factor that ‘does not contain y’ ‘72, Highest wind speed near L Note thats? = 8 & V2 = 8 + 2/7 = (1 V7)? Hence, there are four values of <1 JI1- 9-14 tT , x8 =x) To find the slope, 2’ = 8 — Py’ = AO —CONTINUED— Section 2.5 Implicit Differentiation 189 73, CONTINUED— Forx = =1~ Ji,y' = H{/7 +7) andthe in is na YI + ae 1+ J) +3 =H(VT + 1s + 87 + Forx = 1 J¥,y' = {V7 ~ 7), and the tne is m= HVT = ale 1+ 1) +3= AT ~ 1) + 23 BV} For = -1 + Jiny’= -(J7 ~ 7), and the line is we HVT= Meet = Vi) +3 = -flV7~ I~ (3 ~ 87) Forx= 1+ Ji.y’= -Y(J7 + 7), andthe in is y= YT + Me 1 — VI) +3 = (V7 + Ihe ~ (87 + 23] (©) Equating y, and y4 YV1 et JI) +3 = “HVT + Me +3 (Vi- Met VI) = (V+ Me 1-7) Vint Si 1-141 = Sie = S9- 7412-7 7 16/7 = Nae =87 a7 vera !4 oy set ntns meee (SY) mM Vita ve 75. y= 30/6, p,qintegers and q > 0 418. yeax Ra aya? eee 4% “1 - 3 ee 2a line at - = Be = Tangent line a Gy 9): ¥ = 90 = ~ABO~ 20 a8 abot simeoeye (9 + VEovIe 0) ‘Thus, ify = 24, m= p/a,then y’ = wet. intercept: (0, 5 + V¥eVFo) Sum of intercepts: (x9 + V%eV50) + (90 + VEGVI6) = 0 + 2300 + Yo V+ Vi) VP me 160__Chapter2__ Differentiation 76. a + y= 25, slope = 3 2x + dy’=0 Horizontal tangents at (0, 1) and (0, ~1) Note: y4—y? +2? =0 1s Ie 2 xea3 Points: (3, —4) and (3,4) If you graph these four equations, you will see that these are horizontal tangents at (0, +1), but not at (0,0). 78. (4,0) ah ere oe oe a9 y= 0 y ~ox(x- 4) But, 9x? + 4y2 = 36 m9 4y? = 36 ~ 9:2 Hence, ~91? + 36x = 4 Points on eps: (1,33/3) (3): ‘Consider the slope of the normal line joining (x, 0) and (x, ) = (2,9) on the parabola, ~ty = $= yma

Вам также может понравиться